You are on page 1of 61

https://upscpdf.com/ https://upscpdf.

com/

VISIONIAS
www.visionias.in

Test Booklet Series

TEST BOOKLET

GENERAL STUDIES (P) 2024 – Test – 4157


C
Time Allowed: Two Hours Maximum Marks: 200

INSTRUCTIONS

1. IMMEDIATELY AFTER THE COMMENCEMENT OF THE EXAMINATION, YOU SHOULD CHECK THAT THIS BOOKLET
DOES NOT HAVE ANY UNPRINTED OR TURN OR MISSING PAGES OR ITEMS, ETC. IF SO, GET IT REPLACED BY A
COMPLETE TEST BOOKLET.

2. ENCODE CLEARLY THE TEST BOOKLET SERIES A, B, C OR D AS THE CASE MAY BE IN THE APPROPRIATE PLACE IN
THE ANSWER SHEET.

3. You have to enter your Roll Number on the Test Booklet in the Box
provided alongside. Do NOT write anything else on the Test Booklet.

4. This Test Booklet contains 100 items (Questions). Each item is printed in English. Each item comprises four
responses (answers). You will select the response which you want to mark on the Answer Sheet. In case you
feel that there is more than one correct response with you consider the best. In any case, choose ONLY ONE
response for each item.

5. You have to mark all your responses ONLY on the separate Answer Sheet provided. See direction in the
answers sheet.

6. All items carry equal marks. Attempt all items. Your total marks will depend only on the number of correct
responses marked by you in the answer sheet. For every incorrect response 1/3rdof the allotted marks will be
deducted.

7. Before you proceed to mark in the Answer sheet the response to various items in the Test booklet, you have to
fill in some particulars in the answer sheets as per instruction sent to you with your Admission Certificate.

8. After you have completed filling in all responses on the answer sheet and the examination has concluded, you
should hand over to Invigilator only the answer sheet. You are permitted to take away with you the Test
Booklet.

9. Sheet for rough work are appended in the Test Booklet at the end.

DO NOT OPEN THIS BOOKLET UNTIL YOU ARE ASKED TO DO SO


1 www.visionias.in ©Vision IAS

https://upscpdf.com/ https://upscpdf.com/
https://upscpdf.com/ https://upscpdf.com/

.
1. Which of the following statements best 4. Consider the following statements regarding
describes cropping intensity? the Cham Lama dance:
(a) It refers to using high amount of 1. Guru Rinpoche is regarded as the
fertilizers to increase production of a propounder of this dance tradition.
2. Heavy masks are worn during the
particular crop.
performance of this dance.
(b) It refers to raising of a number of crops
3. It is performed by Buddhist monks in the
from the same field during one Himalayan states.
agriculture year. How many statements given above are
(c) It refers to reserving an area of arable correct?
land for a particular crop. (a) Only one
(d) It refers to increasing net sown area to (b) Only two
increase overall production of grains. (c) All three
(d) None
2. Consider the following statements regarding
5. Consider the following statements regarding
the ‘Energy Transition Accelerator (ETA)’:
invasions in ancient India:
1. It is a new climate finance mechanism 1. Alexander was the first foreign invader
unveiled by the International Energy Agency to invade India.
(IEA) at the UNFCC summit held in Dubai. 2. The Indian king Porus lost his empire to
2. It will allow companies to fund clean Alexander following his defeat at the
energy projects in developing and developed Battle of Hydaspes.
countries and gain carbon credits. Which of the statements given above is/are
Which of the statements given above is/are correct?
(a) 1 only
correct?
(b) 2 only
(a) 1 only
(c) Both 1 and 2
(b) 2 only
(d) Neither 1 nor 2
(c) Both 1 and 2
(d) Neither 1 nor 2 6. Consider the following statements regarding
India’s Indigenous prototype fast breeder
3. In the context of Ashtapradhan, consider the reactor (PFBR):
following statements: 1. It is a fast breeder reactor designed to
1. Ashta Pradhan as an administrative and produce more fissile material than it
consumes.
advisory council was the creation of
2. It uses only natural or low-enriched U-
Shivaji.
238 as the fissile material.
2. Under Shivaji, these offices were
3. Its development led to the beginning of
hereditary and permanent. the final stage of India's three-stage
3. They were directly paid by the Nuclear Program.
exchequer and no jagir was granted to 4. It uses liquid sodium as a coolant in two
any civil or military officer. circuits to transfer heat energy to
How many of the statements given above are generators for producing electricity.
correct? How many of the above statements are
correct?
(a) Only one
(a) Only one
(b) Only two
(b) Only two
(c) All three
(c) Only three
(d) None (d) All four

2 www.visionias.in ©Vision IAS

https://upscpdf.com/ https://upscpdf.com/
https://upscpdf.com/ https://upscpdf.com/

.
7. Consider the following: 10. With reference to the India-Europe Free

1. Minimum Capital Requirements Trade Agreement (EFTA), which of the


following statements given below is correct?
2. Supervisory review of Capital Adequacy
(a) The deal includes agreements with
3. Market Discipline
respect to both goods and services.
How many of the above are the pillars of the (b) Binding commitment of $100 billion
Basel III norms? FDI in next 5 years.

(a) Only one (c) Among EFTA countries, Norway is the


largest trading partner of India.
(b) Only two
(d) The investment covers both FDI and
(c) All three
FPI.
(d) None
11. Recently, the snake-venom case has been in

8. Consider the following statements regarding the news. In this regard, consider the
following statements:
Star Dunes:
1. Snakes produce venom in order to
1. They are Circular-shaped sand formations
immobilize their prey.
found in deserts, with arms stretching out 2. The toxins present in snake venoms are
from a central peak. produced in glands present inside the
mouth of a snake.
2. Strong winds blowing in one direction are
3. Python is an example of a venomous
the primary factor contributing to the
snake.
formation of star dunes. How many of the above statements
Which of the statements given above is/are are correct?
correct? (a) Only one
(b) Only two
(a) 1 only
(c) All three
(b) 2 only
(d) None
(c) Both 1 and 2

(d) Neither 1 nor 2 12. Arrange the following rates in the


increasing order of their values.
1. Reverse Repo Rate
9. Dakshinaranjan Mukherjee, Ram Gopal
2. Repo Rate
Ghose and Krishna Mohan Banerjee are
3. Marginal Standing Facility Rate
members of Select the correct answer using the code
(a) Anushilan Samiti given below.

(b) Young Bengal Group (a) 2-1-3


(b) 1-2-3
(c) Jugantar Group
(c) 1-3-2
(d) Theosophical Society
(d) 3-2-1

3 www.visionias.in ©Vision IAS

https://upscpdf.com/ https://upscpdf.com/
https://upscpdf.com/ https://upscpdf.com/

.
13. Consider the following statements: 16. Consider the following factors:
1. India’s domestic production of crude oil 1. High salinity levels in water
accounts for more than one-third of the 2. High humidity levels
country’s supply needs. 3. High wind speeds
4. High air pressure over the water surface.
2. Mumbai High field is the largest oil field
How many of the above factors decrease the
in India.
rate of evaporation?
3. The Barauni refinery is an example of
(a) Only one
market-based refinery. (b) Only two
How many of the above statements are (c) Only three
correct? (d) All four
(a) Only one
(b) Only two 17. Consider the following statements lithium
(c) All three industry in India:
1. Currently, India imports 100% of its
(d) None
lithium requirements.
2. The bulk of the lithium is sourced from
14. Consider the following statements regarding
the lithium triangle countries of South
the production of pulses in India: America.
1. Pulses account for about one-fifth of the 3. The first-ever lithium exploration and
total area under food grains. mining project by a Government
2. Kharif pulses contribute more than 60 Company of India is being undertaken in
percent of the total production. Argentina.
3. Gram is the most dominant pulse having How many of the above statements are
correct?
a share of more than 40 percent.
(a) Only one
4. India is the largest exporter of pulses.
(b) Only two
How many of the above statements are
(c) All three
correct? (d) None
(a) Only one
(b) Only two 18. Consider the following statements regarding
(c) Only three the Interest Rate Growth Differential
(d) All four (IRGD):
1. It is the difference between interest paid
on government debt and the economy’s
15. Under the current account of the Balance of
nominal growth rate.
Payments, which of the following will not be
2. India has negative IRGD due to very
included under the invisibles in the current
low-interest rates.
account? Which of the statements given above is/are
(a) Remittances correct?
(b) Income earned by MNCs from their (a) 1 only
investment in India (b) 2 only
(c) Software exports (c) Both 1 and 2
(d) NRI deposits (d) Neither 1 nor 2

4 www.visionias.in ©Vision IAS

https://upscpdf.com/ https://upscpdf.com/
https://upscpdf.com/ https://upscpdf.com/

.
19. Consider the following statements: 22. Consider the following statements regarding
1. The children of foreign diplomats posted the landslide susceptibility in India:
in India and enemy aliens cannot acquire 1. The regions of Andaman and Nicobar
Indian citizenship by birth. Islands come under the very high
2. In India, both a citizen by birth as well
vulnerability zone.
as a naturalized citizen are eligible for
2. Trans-Himalayan areas of Ladakh come
the post of President.
under the zone of moderate to low
3. If any foreign territory becomes a part of
vulnerability.
India, the President specifies who among
3. Arunachal Pradesh has the highest
the people of territory shall be the citizen
of India. percentage of land area susceptible to
How many of the above statements are landslides.
correct? How many of the statements given above are
(a) Only one correct?
(b) Only two (a) Only one
(c) All three (b) Only two
(d) None (c) All three
(d) None
20. "Obelisks" recently discovered by scientists
are associated with which of the following?
23. Consider the following:
(a) Newly identified intermediary life forms
1. Phosphate fertilizer manufacture and use
between viruses and viroids.
(b) It is a procedure that involves mimicking 2. Battery waste
the functions of the human brain using 3. Cigarette smoke
an inorganically developed model. 4. Coal combustion
(c) Artifacts discovered in ancient Egyptian How many of the above are the sources of
archaeological sites. cadmium pollution?
(d) Geological formations observed in (a) Only two
remote desert regions. (b) Only three
(c) All four
21. Consider the following statement: (d) None
1. If 450 members are present on a certain
day, a minimum of 301 members votes
24. Consider the following pairs:
are a must for a constitution amendment
Important Iron mines Regions
bill to be passed in Lok Sabha.
2. For impeachment of the President, a 1. Sulaipet : Jharkhand
minimum of 273 members of the Lok 2. Katamati : Odisha
Sabha should vote in support of the 3. Rajhara : Chattisgarh
impeachment. How many of the above pairs are correctly
Which of the statements given above
matched?
is/are not correct?
(a) Only one
(a) 1 only
(b) Only two
(b) 2 only
(c) All three
(c) Both 1 and 2
(d) Neither 1 nor 2 (d) None

5 www.visionias.in ©Vision IAS


https://upscpdf.com/ https://upscpdf.com/
https://t.me/visioniastestseries2024
https://upscpdf.com/ https://upscpdf.com/

.
25. Consider the following statements with 27. This plant, commonly found across Europe
reference to personal income: and Asia, thrives in the Himalayan region of
1. Personal income is the income available India, particularly in the cold deserts of

to persons from all sources to dispose of Ladakh and Spiti, where it grows above the

as they choose. tree line. Its leaves, flowers, seeds, and fruits

2. It is obtained from private income by boast medicinal properties, traditionally used

deducting undistributed profit and to combat ageing. In Ladakh, it holds

corporate tax payable by the enterprise. revered titles such as the 'Wonder Plant' and

Which of the statements given above is/are 'Gold Mine'. This plant significantly
contributes to Ladakh's agricultural
correct?
economy, with 90% of its production
(a) 1 only
originating from the region. It is accorded
(b) 2 only
the Geographical Indication (GI) tag for its
(c) Both 1 and 2
unique soil conservation role.
(d) Neither 1 nor 2
(a) Sea Buckthorn
(b) Ramban Anardana
26. Consider the following statements regarding
(c) Apricot
the Sixth Schedule of the Indian Consitution:
(d) Adi Kekir
1. The inclusion of an area of a state/Union
territory under the Sixth schedule allows
28. Consider the following statements regarding
it to create Autonomous District
the Financial Committees of the Parliament:
Councils (ADCs).
1. The Public Accounts Committee
2. The idea of ADCs has its origins in the
examines the annual audit reports of the
Tribal sub-plan.
CAG.
3. The Sixth Schedule discriminates
2. Members of the Rajya Sabha have no
against non-tribals including placing representation in the Estimates
infringements on fundamental rights. Committee.
4. ADCs in the sixth schedule areas act like 3. The Committee on Public Undertakings
mini-states with powers and was created on the recommendations of
responsibilities of legislature, executive the First Administrative Reforms
and judiciary. Commission.
How many of the statements given above are How many of the statements given above are
not correct? correct?
(a) Only one (a) Only one
(b) Only two (b) Only two
(c) Only three (c) All three

(d) All four (d) None


6 www.visionias.in ©Vision IAS
https://upscpdf.com/ https://upscpdf.com/
https://t.me/visioniastestseries2024
https://upscpdf.com/ https://upscpdf.com/

.
29. Consider the following statements regarding 32. In the context of macroeconomics, the
the Asset Management Company (AMC) in problem of fiscal drag is often ascribed to
India: (a) the failure of the government to collect
1. An AMC is a registered financial enough taxes due to a narrow tax base.
institution that pools funds from multiple (b) very low consumer spending due to
investors and invests in various financial increased taxation.
instruments. (c) the deliberate government policy to
2. AMCs have to follow the asset pursue a low-tax regime.
allocation scheme approved by the (d) raising of interest rates by the central
Securities Exchange Board of India
bank of the country.
(SEBI).
3. The office of the Registrar of Companies
33. With reference to Prevention of Money
under the Ministry of Corporate Affairs
Laundering Act, which of the following
regulates the AMCs in India.
statement is not correct?
How many of the above statements are
(a) Activities related to Virtual Digit
correct?
Assests and Cryptocurrency are included
(a) Only one
under the Act.
(b) Only two
(c) All three (b) PMLA can be invoked merely by citing
(d) None a standalone criminal conspiracy charge
under the Bharatiya Nyaya Sanhita (or
30. The ‘Y20 Summit’, mentioned in the news IPC).
recently, is: (c) PMLA Appellate Tribunal's decision can
(a) an annual conference of National Space be challenged directly in the High
Society Court.
(b) a meeting hosted by World Economic (d) It is the only Act in the country in which
Forum to discuss global industry a statement recorded before an
agendas investigating officer is admissible in
(c) the official group of the G20 countries court as evidence.
with focus on global youth leadership
(d) a biennial informal summit held amongst 34. Consider the following statements regarding
BIMSTEC countries the Home Minister’s Advisory Committee
(HMAC):
31. Which of the following statements best 1. All the five UTs without legislature have
describes the idea of popular sovereignty? the forum of HMAC.
(a) It is a system in which the source of
2. It is chaired by the Administrator of the
authority of the government comes from
concerned UTs.
the people.
3. It discusses the general issues relating to
(b) It is a system where source of authority
social and economic development of the
lies with the parliament elected through
UTs.
free and fair elections.
How many of the above statements are
(c) It is a tool of direct democracy that seeks
correct?
to capture the opinion of people on a
(a) Only one
matter of public policy.
(d) It is a doctrine that states that the final (b) Only two
authority of the State is derived from the (c) All three
Constitution. (d) None
7 www.visionias.in ©Vision IAS
https://upscpdf.com/ https://upscpdf.com/
https://t.me/visioniastestseries2024
https://upscpdf.com/ https://upscpdf.com/

.
35. Who among the following is known as the 38. In the cell of a living organism, anaerobic
guardian of the public purse and controls the respiration mainly takes place
entire financial system of the country? (a) In the mitochondria.
(a) Governor of the RBI (b) Inside the nucleus.

(b) Public Accounts Committee (c) In the cytoplasm.

(c) Chairman of the Finance Commission (d) Inside the vacuoles.

(d) Comptroller and Auditor General


39. Consider the following metals:

36. The Central Ground Water Authority has 1. Cobalt

been constituted under which of the 2. Magnesium

following acts? 3. Copper

(a) Water (Prevention & Control Of 4. Nickel

Pollution) Act, 1974 Which of the metals given above show

(b) Forest (Conservation) Act, 1980 ferromagnetic behavior under normal

(c) Air (Prevention and Control of temperature and pressure conditions?


(a) 1, 2 and 3 only
Pollution) Act, 1981
(b) 1 and 4 only
(d) Environment (Protection) Act, 1986
(c) 2, 3 and 4 only
(d) 3 and 4 only
37. Consider the following statements with
reference to the Animal Welfare Board of
40. Consider the following statements with
India (AWBI):
reference to different types of salts:
1. It is a statutory body under the Ministry
1. Compositionally, pink salt is fifty
of Environment, Forest and Climate
percent sodium chloride and fifty
Change.
percent potassium chloride.
2. It takes care of the welfare of
2. The pungent smell of black salt is due to
domesticated and wild animals in
the presence of excess iodine.
captivity.
3. The pink salt from the Himalayas is
3. Zoos in India are regulated by the
gaining popularity because it is devoid
AWBI.
of any impurities.
Which of the statements given above is/are
How many of the statements given above
correct?
are correct?
(a) 2 only
(a) Only one
(b) 2 and 3 only
(b) Only two
(c) 1 and 3 only
(c) All three
(d) 1, 2 and 3
(d) None

8 www.visionias.in ©Vision IAS


https://upscpdf.com/ https://upscpdf.com/
https://t.me/visioniastestseries2024
https://upscpdf.com/ https://upscpdf.com/

.
41. With reference to the evolution of the 44. In the context of Modern Indian History,
Constitution in India, which of the following Which of the following statements best
amendments/changes happened earliest? describes the 'German Plot' or the
(a) Innovation of Basic Structure Doctrine
'Zimmerman Plan'?
by the Supreme Court
(a) A conspiracy planned by the Jugantar
(b) Mandatory Presidential assent to a
constitution amendment bill party to incite a pan-Indian rebellion
(c) Mandatory Cabinet recommendation for against the British Raj during World
National Emergency War I.
(d) Addition of Fundamental duties to the (b) A British proposal to grant India
constitution
dominion status in exchange for support
during World War II.
42. With reference to the Coastal Regulation
Zone (CRZ), consider the following (c) A diplomatic strategy devised by the

statements: British to suppress nationalist


1. The Shailesh Nayak Committee was movements in India during World War I.
related to the coastal zone regulation.. (d) A clandestine operation orchestrated by
2. CRZ-I is situated between high tide lines Indian nationalist leaders to negotiate
and low tide lines.
independence from British rule during
3. CRZ-II constitutes the developed areas
World War II.
up to the shoreline, which fall within the
existing municipal limits.
4. Demarcation of the hazard line in the 45. Recently, a landlocked water body in the
CRZ is done by the respective state USA, 'Salton Sea' was in the news for which
governments. of the following reasons?
How many of the statements given above are
(a) Increased toxicity due to industrial
correct?
effluents
(a) Only one
(b) Only two (b) Shrinking of the sea due to climate

(c) Only three change


(d) All Four (c) Presence of bacteria with medicinal
properties
43. Which of the following is not correct
(d) Discovery of massive lithium deposits
regarding the National Savings Certificate
(NSC)?
46. The right to free legal aid, to any person in
(a) National Savings Certificate is a fixed-
income post office savings scheme. India, is a fundamental right under:
(b) Non-resident Indians (NRI) cannot (a) Right against exploitation
purchase NSC certificates. (b) Right to protection against arrest and
(c) NSC offers tax benefits without any detention
limits.
(c) Right to freedom of speech
(d) Banks and NBFCs accept NSC as
(d) Right to life
collateral or security for secured loans.

9 www.visionias.in ©Vision IAS


https://upscpdf.com/ https://upscpdf.com/
https://t.me/visioniastestseries2024
https://upscpdf.com/ https://upscpdf.com/

.
47. Consider the following pairs: 50. Consider the following statements regarding
River Location Peer-To-Peer (P2P) Lending:
1. Ib River : Odisha
1. P2P lending websites connect borrowers
2. Falgu River : Bihar
to lenders cutting out the financial
3. Pavana River : Maharashtra
institution as the middleman.
How many of the above given pairs are
correctly matched? 2. Investing money through peer-to-peer
(a) Only one lending is safer than putting money in
(b) Only two savings accounts.
(c) All three
3. Currently, P2P lending is not regulated
(d) None
in India.
How many of the above statements are
48. With reference to Medical Termination of
Pregnancy (MTP) Act, 2021, consider the correct?
following statements: (a) Only one
1. Unmarried women cannot undergo (b) Only two
pregnancy termination under this Act.
(c) All three
2. Termination up to 20 weeks requires a
(d) None
single registered medical practitioner's
advice.
3. Pregnancy termination is not allowed 51. Consider the following statements regarding
beyond 24 weeks in any case. the International Network of Solar
How many of the statements given above are Technology and Application Resource
correct?
Centres (Project STAR C):
(a) Only one
1. It is an initiative of International
(b) Only two
Renewable Energy Agency.
(c) All three
(d) None 2. It seeks to to create a strong network of
institutional capacities within
49. Consider the following pairs: International Solar Alliance Member
Book Author States.
1. Majma-ul-Bahrain : Dara Shikoh
3. Its first phase will be focused on South
: Abdul Hamid
2. Padshahnama Asian region.
Lahori
How many of the above statements are
3. Nuh Siphir : Amir Khusrau
correct?
How many of the pairs given above are
correctly matched? (a) Only one
(a) Only one (b) Only two
(b) Only two (c) All three
(c) All three (d) None
(d) None

10 www.visionias.in ©Vision IAS


https://upscpdf.com/ https://upscpdf.com/
https://t.me/visioniastestseries2024
https://upscpdf.com/ https://upscpdf.com/

.
52. Consider the following statements regarding 55. Which one of the following is the primary
Indian puppetry: aim of CRISPR-based approaches in the
1. Kundhei from Odisha is a form of glove context of sickle cell disease?
puppetry. (a) To improve red blood cell flexibility
2. Rod puppetry is mostly found in West (b) To correct the underlying genetic
Bengal and Odisha. mutation
3. Tholu Bommalata is a form of string (c) To induce further genetic mutations
puppetry from Andhra Pradesh. (d) To enhance hemoglobin production
How many of the statements given above are
correct?
56. Consider the following statements:
(a) Only one
1. Full faith and credit is to be given
(b) Only two
throughout the territory of India to
(c) All three
public acts, records and judicial
(d) None
proceedings of the Centre and every
state.
53. Consider the following:
2. Final judgments and orders of civil and
1. SIDBI
criminal courts in any part of India are
2. Port Trusts
capable of execution anywhere within
3. Limited Liability Partnership (LLP)
India.
How many of the following are not eligible
Which of the statements given above is/are
to raise funds under External Commercial
correct?
Borrowings (ECBs) in India?
(a) 1 only
(a) Only one
(b) 2 only
(b) Only two
(c) All three (c) Both 1 and 2

(d) None (d) Neither 1 nor 2

54. Which of the following features of the 57. By which one of the following acts did the

Indian constitution strives to implement the British India government grant the voting

axiom of ‘government of laws and not of rights to Indian women for the first time in

men’? the provinces?

(a) Preamble (a) Government of India Act, 1935


(b) Affirmative action policies (b) Montagu-Chelmsford Reforms 1919
(c) Fundamental Rights (c) Indian Councils Act 1892
(d) Directive Principles of States Policy (d) Morley Minto reform 1909

11 www.visionias.in ©Vision IAS


https://upscpdf.com/ https://upscpdf.com/
https://t.me/visioniastestseries2024
https://upscpdf.com/ https://upscpdf.com/

.
58. Consider the following statements regarding 61. Which of the following is an example of a
the National Games of India: foundation species?
1. The first National Games on the lines of (a) A beaver building a dam that creates a
the Olympics was held in Lucknow. pond
2. It is organized under the jurisdiction of (b) A top predator regulating the population
the Sports Authority of India. of prey species
3. The recent edition of these games was (c) An invasive species outcompeting native
held in Goa. vegetation
How many of the statements given above are (d) A decomposer breaking down organic
correct? matter in soil
(a) Only one
(b) Only two 62. Consider the following statements regarding
(c) All three Indian Ocean Tuna Commission :
(d) None 1. It is an intergovernmental organisation
responsible for the management of tuna
and tuna-like species in the Indian
59. Consider the following statements with
Ocean.
reference to the NAMASTE scheme:
2. India is a member of this organization.
1. It is a joint initiative of the Ministry of
3. Tuna fish are endemic to Indian Ocean.
Social Justice and Empowerment and the
How many of the above statements are
Ministry of Skill Development and
correct?
Entrepreneurship to ensure safety and
(a) Only one
dignity of sanitation workers.
(b) Only two
2. It envisages collectivisation of sanitation
(c) All three
workers into Self Help Groups (SHGs).
(d) None
3. It will provide 100% capital subsidies
for procurement of sanitation related
63. Which of the following statements best
equipment.
describes the 'Article 99' of the UN Charter?
How many of the statements given above are
(a) Discretionary power given to the UN
correct? Secretary-General to bring to the
(a) Only one attention of the UN Security Council
(b) Only two (UNSC) any matter which may threaten
(c) All three the maintenance of international peace
(d) None and security
(b) Representatives of the Members of the
60. Consider the following pairs: United Nations and officials of the
Festivals State Organization shall enjoy privileges and
1. Ker Puja : Uttar Pradesh immunities necessary for the
2. Tamladu : Telangana independent exercise of their functions
3. Nua Khai : Odisha (c) Development of pacific settlement of
How many of the pairs given above are local disputes through regional
correctly matched? arrangements
(a) Only one (d) Guarantee that Jews had the unalterable
right to live anywhere in Palestine, in the
(b) Only two
region between the Jordan River and the
(c) All three
Mediterranean Sea
(d) None

12 www.visionias.in ©Vision IAS


https://upscpdf.com/ https://upscpdf.com/
https://t.me/visioniastestseries2024
https://upscpdf.com/ https://upscpdf.com/

.
64. Consider the following statements about the 67. Consider the following statements about the
Participatory Guarantee Scheme (PGS) for Wildlife Protection Amendment Act of
India: 2022:
1. It is a quality assurance initiative for the 1. The amendment has added a new
certification of organic produce. Schedule IV for the animals listed in the
2. It is implemented by the Ministry of Appendices of CITES.
Commerce and Industry. 2. Schedule I consists of animals like
3. PGS involves third-party certification of tigers, which are given the utmost
organic produce. protection.
How many of the statements given above are 3. Certain plant varieties are protected
correct? under Schedule III.
How many of the above statements are
(a) Only one
correct?
(b) Only two
(a) Only one
(c) All three
(b) Only two
(d) None
(c) All three
(d) None
65. Consider the following statement regarding
sandstone:
68. Consider the following statements about the
1. It is a minor mineral as per the Mines
PM Surya Ghar Muft Bijli Yojna:
and Minerals (Development and 1. It is a Central Scheme that aims to
Regulation) Act, 1957. provide free electricity to one crore
2. It is a metamorphic rock. households in India.
3. In India, it is found only in Rajasthan 2. The scheme provides for a subsidy of
and Madhya Pradesh. 100% of the solar unit cost for systems
How many of the statements given above are up to 2 kW capacity.
correct? 3. Net metering is not important for the
(a) Only one functioning of the scheme.
(b) Only two How many of the above statements are
(c) All three correct?
(d) None (a) Only one
(b) Only two
66. Which of the following statements best (c) All three
describes the role of 'Khutba' in medieval (d) None
history?
(a) A tax levied by the ruler on his subjects 69. Consider the following statements:
for the maintenance of the royal 1. Rifting refers to the tearing apart of a
treasury. single tectonic plate into two are more.
(b) A diplomatic correspondence exchanged 2. The east African rift is an ongoing
between two sovereigns to foster process at the borders of the African
alliances. Nubian, African Somali, and Arabian
plates.
(c) A sermon delivered publicly to
Which of the statements given above is/are
symbolize the Islamic ruler's authority
correct?
and legitimacy.
(a) 1 only
(d) An architectural style developed under
(b) 2 only
the patronage of medieval rulers for
(c) Both 1 and 2
constructing administrative buildings.
(d) Neither 1 nor 2

13 www.visionias.in ©Vision IAS


https://upscpdf.com/ https://upscpdf.com/
https://t.me/visioniastestseries2024
https://upscpdf.com/ https://upscpdf.com/

.
70. Consider the following statements with 73. Consider the following pairs:
reference to the ‘Multi-Angle Imager for Tribal Leader Associated
Aerosols mission' (MAIA), recently seen in
Movement
the news :
1. It was launched by the National 1. Chakra Bisoi : Santhal Rebellion
Aeronautics and Space Administration 2. Tomma Dora : Koya Rebelllion
(NASA) and the Italian Space Agency
3. Sido and Kanhu : Khond Uprisings
(ASI) to investigate the health impacts
of air pollution. How many of the above pairs are correctly
2. The mission utilizes a constellation of matched?
miniature satellites for high-resolution (a) Only one
imaging of aerosols in selected cities. (b) Only two
3. New Delhi is among the primary target
(c) All three
cities identified for the mission.
How many of the statements given above are (d) None
correct?
(a) Only one 74. Consider the following statements about the
(b) Only two trends in urbanization in India:
(c) All three
1. The urbanization in India has steadily
(d) None
increased for the last two decades.
71. Consider the following pairs: 2. Urbanization is primarily driven by the
Newspaper Leader growth of the secondary sector.
: James Augustus 3. There is a growing trend of counter-
1. India Gazette
Hicky
urbanization in India.
2. Sambad Kaumudi : Dwarkanath Tagore
How many of the above given statements are
: Girishchandra
3. Hindu Patriot correct?
Ghosh
How many pairs given above are correctly (a) Only one
matched? (b) Only two
(a) Only one (c) All three
(b) Only two
(d) None
(c) All three
(d) None
75. With reference to 2019 Amendment to
72. With reference to the United Nations Foreign (Tribunals) Order, 1964, consider
Country Team (UNCT), consider the the following statements:
following statements:
1. The powers to constitute tribunals is
1. It includes all UN entities working
on sustainable development in vested with the Centre
programme countries. 2. The amended order also empowers
2. It is led by the UN Resident individuals to approach the Tribunals
Coordinator in respective countries. 3. Foreigners Tribunals under this order
3. The UN Sustainable Development
have been established in all the states
Group oversees the coordination of
UNCT. How many of the statements above are
How many of the statements above is/are correct?
correct? (a) Only one
(a) Only one
(b) Only two
(b) Only two
(c) All three (c) All three
(d) None (d) None

14 www.visionias.in ©Vision IAS


https://upscpdf.com/ https://upscpdf.com/
https://t.me/visioniastestseries2024
https://upscpdf.com/ https://upscpdf.com/

.
76. Consider the following statements regarding 79. In the context of biotechnology, prime
the International Space Station (ISS):
editing can be best described as:
1. It is the first-ever space station built in
Earth's Orbit, with the collaboration of (a) A method for analyzing gene expression
five nations. levels in cells.
2. It is the only space station operating in
(b) A technique for editing DNA sequences
the Earth's Orbit.
3. Rakesh Sharma is the first Indian with high precision.
astronaut to go to space and board the
(c) A protocol for amplifying DNA
ISS
How many of the above statements are fragments in the laboratory.
correct? (d) A system for predicting protein
(a) Only one
(b) Only two structures based on genomic data.
(c) All three
(d) None
80. In the parlance of macroeconomics, what do

77. Consider the following statements regarding you understand by 'Arbitrage'?


Delhi Manifesto, 1929: (a) It is the buying and selling of assets in
1. It was released by Muslim League in
1929 to counter the Nehru Report before different markets in order to take
First Round Table conference. advantage of differing prices for the
2. It demanded that equal representation
same asset.
should be given to Muslims League and
Congress in Round Table conferences. (b) It is the cost borne by the central bank in
Which of the statements given above is/are printing currency notes.
correct?
(a) 1 only (c) It is the fee paid by investors and traders
(b) 2 only as a percentage of their net gains to the
(c) Both 1 and 2
trading platforms.
(d) Neither 1 nor 2
(d) It is the cost borne by the government in
78. Consider the following statements regarding tax collections in an an economy.
solar manufacturing:
1. India is the second-largest module
manufacturing country in the world. 81. In which of the following groups are all the
2. Gujarat is the leading state in India in
four countries members of Indian Ocean
terms of upcoming PV module
manufacturing capacity. Rim Association?
3. India's solar PV manufacturing industry
(a) Australia, Bangladesh, Comoros and
is heavily reliant on China for raw
materials. Pakistan
How many of the above statements are (b) India, Indonesia, Kenya and Somalia
correct?
(c) Madagascar, South Africa, Sri Lanka
(a) Only one
(b) Only two and Saudi Arabia
(c) All three
(d) Oman, Seychelles, UAE and New
(d) None
Zealand

15 www.visionias.in ©Vision IAS


https://upscpdf.com/ https://upscpdf.com/
https://t.me/visioniastestseries2024
https://upscpdf.com/ https://upscpdf.com/

.
82. Consider the following countries: 85. Recently, researchers at the Geological
1. Democratic Republic of Congo Survey of India discovered Vanadium from

2. Ghana Gulf of Khambat. With reference to this,

3. Mali consider the following:


1. Energy storage
4. Nigeria
2. Nuclear reactors
How many of the above African countries
3. Steel manufacturing
do not have a coastline along the South
4. Aerospace components
Atlantic Ocean?
How many of the above are applications of
(a) Only one vanadium?
(b) Only two (a) Only one
(c) Only three (b) Only two
(d) All four (c) Only three
(d) All four

83. Consider the following statements regarding


the ISRO's CE-20 rocket engine: 86. Which of the statements given below
is not correct regarding the Basholi painting?
1. It uses liquid hydrogen, which is the
(a) It is a distinctive form of mural
most efficient fuel for rockets.
painting.
2. It is the first indigenous cryogenic
(b) It is the first GI-tagged handicraft from
engine developed by ISRO.
the Jammu region.
3. It was recently human-rated and can be (c) It belongs to the Pahari school of
used in GSLV Mk III launch vehicles for paintings.
Gaganyaan missions. (d) Natural dyes and pigments are used in
How many of the above statements are these paintings.
correct?
(a) Only one 87. Consider the following statements:

(b) Only two 1. A money bill is a financial bill certified


by the President.
(c) All three
2. All Finance bills that are not money bills
(d) None
must include recommendations of the
Rajya Sabha.
84. In the Rig Vedic period, the term "Yava"
Which of the statements given above is/are
referred to:
correct?
(a) Deities in warfare (a) 1 only
(b) Musical instrument (b) 2 only
(c) Ornaments (c) Both 1 and 2
(d) Food grains (d) Neither 1 nor 2

16 www.visionias.in ©Vision IAS


https://upscpdf.com/ https://upscpdf.com/
https://t.me/visioniastestseries2024
https://upscpdf.com/ https://upscpdf.com/

.
88. Consider the following pairs: 91. Consider the following pairs:
Conspiracy Case Associated National Park River passing
Leaders through it
1. Delhi Conspiracy : Barindra Kumar 1. Dibru-Saikhowa : Lohit
Case Ghosh National Park
2. Alipore Conspiracy : Rashbehari Bose 2. Great Himalayan : Sainj
Case National Park
3. Nashik Conspiracy : V.D. Savarkar 3. Silent Valley National : Kabini
Case Park
4. Kanpur Conspiracy : S.A Dange How many of the above pairs are correctly
Case matched?
How many of the above pairs are correctly (a) Only one
matched? (b) Only two
(a) Only one (c) All three
(b) Only two (d) None
(c) Only three
(d) All four 92. With reference to prevailing legal systems,
which of the following best describes the
89. Consider the following statements regarding term 'Common Law'?
public goods: (a) Fundamental law od the country that is
1. Public goods are goods with applicable to both citizens and aliens
characteristics of non-rivalry and non- (b) A set laws that is equally applicable to
excludability. all religious communities within a
2. These goods have zero opportunity cost territory
to society. (c) Laws created by the elected legilative
Which of the statements given above is/are bodies
correct? (d) A body of law that is based on court
(a) 1 only decision or legal precedents
(b) 2 only
(c) Both 1 and 2 93. Which of the following statements regarding
(d) Neither 1 nor 2 phytoremediation is not correct?
(a) Phytoremediation is a cost-effective and
90. Consider the following pairs : environmentally friendly approach to
Power plant State remove pollutants from soil, water, or
1. Sipat Super Thermal : Jharkhand air.
Power Plant (b) Certain plants, called
2. Tiroda Thermal Power : Odisha hyperaccumulators, have the ability to
Plant absorb and accumulate high
3. Talcher Super : Chattisgarh concentrations of heavy metals.
Thermal Power Plant (c) Phytoremediation is a rapid process that
How many of the pairs given above are can completely eliminate all types of
correctly matched? pollutants within a short period.
(a) Only one (d) Phytoremediation can be used to
(b) Only two remediate contaminated sites, such as
(c) All three industrial areas and mining sites.
(d) None

17 www.visionias.in ©Vision IAS


https://upscpdf.com/ https://upscpdf.com/
https://t.me/visioniastestseries2024
https://upscpdf.com/ https://upscpdf.com/

.
94. Consider the following pairs: 97. Consider the following plant species:
Desert Primary Aridity 1. Patwa
Factor 2. Himalayan fritillary
1. Gobi Desert : Continentality 3. Salampanja
2. Atacama Desert : Coastal Cooling How many of the above-mentioned species
3. Mojave Desert : Rainshadow desert are classified as medicinal plants?
How many of the above pairs are correctly (a) Only one
matched? (b) Only two
(a) Only one (c) All three
(b) Only two (d) None
(c) All three
(d) None 98. Consider the following statements regarding
the Processed Steel Slag road:
95. Consider the following species:
1. The construction cost of the processed
1. Ghol or Black spotted Crocker
steel slag road is higher than that of
2. Baobabs
bitumen roads.
3. Eurasian Otter
2. The processed steel slag road will have
4. Fish Mint
thicker layers than regular roads.
How many of the species listed above are
3. Surat has become the first city in India
capable of thriving in aquatic environments?
to implement a processed steel slag road
(a) Only one
project.
(b) Only two
(c) Only three How many of the above statements are

(d) All four correct?


(a) Only one
96. Consider the following statements regarding (b) Only two
Lachit Borphukan: (c) All three
1. Lachit Borphukan was prime minister of (d) None
the Ahom Kingdom of Assam.
2. He was a contemporary of the Mughal 99. Consider the following events:
emperor Aurangzeb. 1. Separation of Burma from India
3. Lachit Borphukan led the Ahom forces 2. All Parties Conference in 1928
to victory against the Mughals in the 3. Dandi March by Mahatma Gandhi
Battle of Saraighat. How many of the above events occurred
Which of the statements given above is/are
during the tenure of Lord Irwin?
correct?
(a) Only one
(a) 1 only
(b) Only two
(b) 2 and 3 only
(c) All three
(c) 1 and 3 only
(d) None
(d) 3 only

18 www.visionias.in ©Vision IAS


https://upscpdf.com/ https://upscpdf.com/
https://t.me/visioniastestseries2024
https://upscpdf.com/ https://upscpdf.com/

.
100. From the decline of the Mauryan dynasty
until the rise of the Gupta Empire, which of
the following kingdoms were holding power
in different parts of India?
1. The Sungas in Magadha
2. The Vardhana dynasty of Thanesar
3. The early Cholas
4. The Satavahanas of Andhra
5. The Palas of Bengal
Select the correct answer using the code
given below
(a) 1, 2 and 4 only
(b) 1, 3, 4 and 5 only
(c) 2, 3 and 5 only
(d) 1, 3 and 4 only

Copyright © by Vision IAS


All rights are reserved. No part of this document may be reproduced, stored in a retrieval system or
transmitted in any form or by any means, electronic, mechanical, photocopying, recording or otherwise,
without prior permission of Vision IAS.

19 www.visionias.in ©Vision IAS


https://upscpdf.com/ https://upscpdf.com/
https://t.me/visioniastestseries2024
https://upscpdf.com/ https://upscpdf.com/

.
VISIONIAS
www.visionias.in
ANSWERS & EXPLANATIONS
GENERAL STUDIES (P) TEST – 4157 (2024)

Q 1.B
• There are only two ways to satisfy the increasing food and other agricultural demands of the country's
rising population: either expanding the net area under cultivation or intensifying cropping over the
existing area.
• The net sown area of the country has risen by about 20 percent since independence and has reached a
point where it is not possible to make any appreciable increase.
• Thus; raising the cropping intensity is the only viable option left. Cropping intensity refers to raising of
a number of crops from the same field during one agriculture year. It can be expressed as
cropping intensity = (Gross cropped area / Net sown area) x 100
• It increases the total output per unit area of land from all crops grown over one agricultural year by
increasing land-use intensity. The advantage is that along with increased output from limited land, it also
increases the demand for labor significantly. For a land-scarce but labor abundant country like India,
a high cropping intensity is desirable not only for fuller utilization of the land resource but also for
reducing unemployment in the rural economy.
• Hence option (b) is the correct answer.

Q 2.D
• Energy Transition Accelerator (ETA): It was launched collaboratively by three entities, The United
States Department of State, The Bezos Earth Fund and the Rockefeller Foundation. Hence,
statement 1 is not correct.
• About Energy Transition Accelerator (ETA):
o Focuses on Emerging Economies: The initiative targets developing and emerging economies to help
them transition from fossil fuels to cleaner energy sources. Hence, statement 2 is not correct.
o Finance via Carbon Credits: ETA creates a carbon credit system. Companies that want to offset their
emissions can buy these credits from participating countries. The credits are generated based on
verified emissions reductions achieved in the participating countries' power sectors.
o Funding Clean Energy Projects: The money from carbon credit sales will be used to fund clean energy
projects in developing countries. This will help them reduce their reliance on fossil fuels and invest in
renewable energy sources.
o Just Transition: One of the core goals of ETA is to ensure a "just transition" for developing countries.
This means that the shift to clean energy should be done in a way that protects jobs and livelihoods.

Q 3.A
• Ashta Pradhan, the administrative and advisory council set up by the Indian Hindu Maratha leader Shivaji
(died 1680), which contributed to his successful military attacks on the Muslim Mughal Empire and to the
good government of the territory over which he established his rule. But, the ashtapradhan was neither the
creation of Shivaji nor was at first organized at, the time of his coronation. The Peshwa, mazumdar,
wakins, dabir, surnis, and the sarnobat also existed under the Deccani rulers. Hence statement 1 is not
correct.
• The king was the pivot of the government and was assisted by a council of ministers called the
Ashtapradhan. Each one was directly responsible to Shivaji and headed a department. Under Shivaji, these
offices were neither hereditary nor permanent and were also frequently transferred. However, under
Peshwas, the posts were made permanent and hereditary. Hence statement 2 is not correct.
• The Ashta Pradhan was designed to encompass all the primary administrative functions of the state, with
each minister being given charge of one role in the administration. Ministerial designations were drawn
from the Sanskrit language. The Ashtapradhan officers of the Maratha Empire received direct payment
1 www.visionias.in ©Vision IAS
https://upscpdf.com/ https://upscpdf.com/
https://t.me/visioniastestseries2024
https://upscpdf.com/ https://upscpdf.com/

.
from the treasury without being granted any land grants (jagirs) i.e. They were directly paid from the
exchequer, and no jagir was granted to any civil or military official. Hence statement 3 is correct.

Q 4.C
• According to Tibetans, Padma Sambhava also known as Guru Rinpoche (Precious Guru) started
the Cham lama dance tradition. That is why Cham dance often depicts scenes of Padmasambhava’s
life. The Guru Dance is performed on June 10 of Tibetan calendar to commemorate the day of his
enlightenment. Monks perform the dance in masks and costumes of the 8 manifestations of Guru
Rinpoche. In their hands, they hold various special instruments that Guru Rinpoche used.. Hence,
statement 1 is correct.
• The dance in itself is seen as a form of meditation and is mostly practiced during festivals like Losar in
Buddhist monasteries. It is a masked dance unique to Tibetan Buddhism. Cham, or the lama dance, is
a form of meditation to liberate the minds of those witnessing the dance. It is a process of awakening the
mind from negative thoughts and leading it towards awakening. Hence, statement 2 is correct.
• The dance ritual of Cham is till date practiced in Himalayas, especially in places where Tibetan
Buddhism prospered in India- Ladakh, Dharamshala, Lahaul Valley, Spiti Valley & Sikkim, and
Bhutan and Tibet. Hence, statement 3 is correct.

Q 5.D
• Alexander's invasion of India occurred during the reign of King Porus in Kashmir and Dana Nanda was
the ruler of the Nanda dynasty in Magdha, not Chandragupta Maurya. Chandragupta Maurya founded the
Maurya Empire in 322 BC, several years after Alexander's invasion.
• In the first half of the sixth century BC, there were several small tribal states in northwest India. There
was no sovereign power to unite these warring tribes. The Achaemenid rulers of Persia or Iran took
advantage of the political disunity of this region. Cyrus, the founder of the Achaemenid dynasty, and the
first invasion of India occurred during the period of Darius I. In 516 BCE, Darius I embarked on a
campaign to Central Asia, conquering the lands surrounding the Indus River. Darius I annexed
parts of Punjab and Sindh. It was believed to be the most fertile and populous part of the
Achaemenid empire. Hence statement 1 is not correct.
• The Battle of Hydaspes was a decisive fought between the armies of Alexander the Great and King Porus
(also known as Poru, Paurava) in 326 BC. It ended in a victory for Alexander's Macedonian Army.
Following the battle, King Porus was brought before Alexander and was asked how he wished to be
treated, Porus replied “Treat me as a king would treat another king”. Impressed, Alexander indeed treated
him like a king, allowing him to retain his lands. Hence statement 2 is not correct.

Q 6.B
• Prototype fast breeder reactor (PFBR):
o The PFBR is a machine that produces more nuclear fuel than it consumes. Hence statement 1 is
correct.
o Its core-loading event is being hailed as a “milestone” because the operationalization of the PFBR
will mark the start of stage II of India’s three-stage nuclear power program. Hence statement 3
is not correct.
• India’s three-stage nuclear power program:
o First stage:
▪ In the first, India used pressurized heavy water reactors (PHWRs) and natural uranium-238
(U-238), which contain minuscule amounts of U-235, as the fissile material.
▪ In nuclear fission, the nucleus of an atom absorbs a neutron, destabilizes, and breaks into two
while releasing some energy. If the destabilized nucleus releases more neutrons, the reactor’s
facilities will attempt to use them to instigate more fission reactions.
▪ The heavy water in PHWR – water molecules containing the deuterium isotope of hydrogen –
slows neutrons released by one fission reaction enough to be captured by other U-238 and U-235
nuclei and cause new fission. The heavy water is pressurized to keep it from boiling. The
reactions produce plutonium-239 (Pu-239) and energy.
o Stage -II:
▪ Only U-235, not U-238, can sustain a chain reaction but it is consumed fully in stage I.
▪ In stage II, India will use Pu-239 together with U-238 in the PFBR to produce energy, U-
233, and more Pu-239. Hence statement 2 is not correct.
▪ The Department of Atomic Energy (DAE) set up a special-purpose vehicle in 2003 called
Bharatiya Nabhikiya Vidyut Nigam, Ltd. (BHAVINI) to implement stage II.
2 www.visionias.in ©Vision IAS
https://upscpdf.com/ https://upscpdf.com/
https://t.me/visioniastestseries2024
https://upscpdf.com/ https://upscpdf.com/

.
o Stage-III:
▪ In stage III, Pu-239 will be combined with thorium-232 (Th-232) in reactors to produce energy
and U-233.
▪ Homi J. Bhabha designed the three-stage program because India hosts roughly a quarter of the
world’s thorium. The three stages are expected to allow the country complete self-sufficiency in
nuclear energy.
• The PFBR utilizes a liquid sodium cooling system with two separate loops. This system transfers heat
generated by the reactor core to a secondary loop containing water, which ultimately drives the turbines
for electricity generation. Hence statement 4 is correct.

Q 7.C
• Basel III reforms are the response of the Basel Committee on Banking Supervision (BCBS) to improve
the banking sector’s ability to absorb shocks arising from financial and economic stress, whatever the
source, thus reducing the risk of spill over from the financial sector to the real economy.
• The Basel capital adequacy framework rests on the following three mutually- reinforcing pillars:
o Pillar 1: Minimum Capital Requirements - which prescribes a risk-sensitive calculation of capital
requirements that, for the first time, explicitly includes operational risk in addition to market and
credit risk.
o Pillar 2: Supervisory Review Process (SRP) - which envisages the establishment of suitable risk
management systems in banks and their review by the supervisory authority.
o Pillar 3: Market Discipline - which seeks to achieve increased transparency through expanded
disclosure requirements for bank
• Hence option (c) is the correct answer.

Q 8.A
• Star dunes are circular-shaped sand formations found in deserts, characterized by their vaguely
pyramid-shaped appearance with arms stretching out from a central peak. Hence statement 1 is correct.
• These geological features, reaching heights of up to about 1,000 feet (300 meters), are among the wonders
of Earth's deserts, contributing to the diverse landscapes of arid regions.
• Location and Description:
o One notable star dune studied by scientists is located in eastern Morocco, within the Sahara Desert,
named Lala Lallia, meaning "highest sacred point" in the local Berber language.
o This star dune rises approximately 330 feet (100 meters) above the surrounding dunes and spans
approximately 2,300 feet (700 meters) wide, containing about 5-1/2 million metric tons of sand.
• Formation and Age:
o These are formed in areas with complex wind regimes, where winds blow from different directions,
and net sand accumulation, points within the desert where big piles of sand can be blown around to
form giant dunes. Hence statement 2 is not correct.
o A groundbreaking study focused on the Lala Lallia star dune provided insights into its internal
structure and formation process.
o Using ground-penetrating radar and luminescence dating techniques, researchers determined that Lala
Lallia took about 900 years to form, accumulating roughly 6,400 metric tons of sand annually as wind
relentlessly blows sand through the desert.
• Characteristics and Significance:
o Star dunes make up just under 10% of the dunes in Earth's deserts and are the tallest ones, surpassing
other types such as crescent-shaped barchan dunes and straight and lengthy linear dunes.
o These geological wonders have also been observed on other celestial bodies, including Mars and
Saturn's large moon Titan.
o The study of star dunes provides valuable insights into desert geomorphology and the dynamics of
wind-driven sediment transport.
• Global Distribution:
o Earth's largest star dunes are found in the Badain Jaran desert in western China.
o Namib Sand Sea in Namibia, large sand seas in Algeria such as the Grand Erg Oriental and Grand Erg
Occidental, and Rub' al Khali in Saudi Arabia.
o In North America, Great Sand Dunes National Park in Colorado contains a series of them.

3 www.visionias.in ©Vision IAS


https://upscpdf.com/ https://upscpdf.com/
https://t.me/visioniastestseries2024
https://upscpdf.com/ https://upscpdf.com/

.
• The Lala Lallia star dune in Morocco is 100m high.

Q 9.B
• 'Young Bengal Group:
o There was a trend of socio-religious reform movements where there was an attempt to create symbols
of change through non-conformist individual activity. This was limited to the 'Derozians' or 'Young
Bengal' who represented a radical stream within the reform movement.
o The members of this group prominent of them being Dakshinaranjan Mukherjee, Ram Gopal
Ghose, and Krishna Mohan Banerjee, stood for a rejection of tradition and revolt against accepted
social norms. Hence option (b) is the correct answer.
o They were highly influenced by "the regenerating new thought from the West" and displayed an
uncompromisingly rational attitude towards social problems.
• Ramgopal Ghosh:
o Ramgopal Ghosh was a leader of the Young Bengal Group, a successful businessman, orator, and
social reformer. He is called the 'Demosthenes of India'. Ghosh was one of the personalities
who helped John Elliot Drinkwater Bethune establish a girls' school in Calcutta.
o Ramgopal Ghosh received his early education at the Hare Preparatory School and joined the Hindu
College in 1824 where he studied under Henry Louis Vivian Derozio.
o Ram Gopal Ghose expressed the rationalist stance of the young Bengal group when he declared: "He
who will not reason is a bigot, he who cannot is a fool and he who does not is a slave".
• Krishna Mohan Banerjee:
o Krishna Mohan Banerjee was a 19th-century Indian thinker who attempted to rethink Hindu
philosophy, religion, and ethics in response to the stimulus of Christian ideas.
o He himself became a Christian and was the first president of the Bengal Christian Association,
which was administered and financed by Indians. He was a prominent member of Henry Louis
Vivian Derozio's (1808–1831) Young Bengal group, educationist, linguist, and Christian
missionary.
• Raja Dakshinaranjan Mukherjee:
o He was the talukdar of Shankarpur in the United Provinces, a social reformer and editor.
o He came under the influence of Henry Louis Vivian Derozio and became a member of the
Young Bengal group. He was a friend of David Hare and helped him in his social
projects. Bethune School was built on land donated by him. He also supported the school in
other ways.
o He edited several journals from a young age and gave shelter to Krishna Mohan Banerjee, who had
caused outrage in Bengali society by converting to Christianity.

Q 10.A
• India-EFTA Pact: Trade and Economic Partnership Agreement (TEPA)
• The Agreement was signed on March 10, 2024
• The agreement involves India and EFTA member states, which are Iceland, Liechtenstein, Norway, and
Switzerland.
• The Goals of the agreement are:
o Promoting investments and exports, particularly in India’s IT, audio-visual sectors, and skilled
professional movement.
o Investment target: The EFTA states shall aim to increase FDI into India by $50 billion within 10
years, and another $50 billion in the five years thereafter. Hence option (b) is not correct.

4 www.visionias.in ©Vision IAS


https://upscpdf.com/ https://upscpdf.com/
https://t.me/visioniastestseries2024
https://upscpdf.com/ https://upscpdf.com/

.
• Additional provisions:
o For the first time, the FTA also included a chapter on commitments to human rights and sustainable
development.
o The agreement will come into force after ratification by the EFTA states, expected possibly by the end
of the year.
o The deal covers various aspects such as trade in goods, rules of origin, intellectual property
rights, services, investment promotion, government procurement, technical trade barriers, and
trade facilitation. Hence option (a) is correct.
o The trade deal also covers gender and environmental aspects.
• It would largely come from provident funds in EFTA countries. These include Norway’s $1.6 trillion
sovereign wealth fund, the world’s largest pension fund, which posted a record profit of $213 billion in
2023 due to its investments in technology stocks. However, it may not be legally binding and falls
under “investment promotion”.
• Excludes Provisions: India and the EFTA also agreed to largely keep “sensitive” farm products and gold
imports out of the pact. Sectors such as dairy, soya, coal and sensitivity related to PLI in sectors such as
pharma, medical devices & processed food, etc. are kept in the exclusion list.
• Among EFTA countries, Switzerland is the largest trading partner of India followed by Norway. It
is the 12th largest investor in India. Hence option (c) is not correct.
• The investment under the agreement covers only FDI and not FPI. Hence option (d) is not correct.
• The EFTA is the intergovernmental organization of Iceland, Liechtenstein, Norway, and
Switzerland. It was set up in 1960 (by the Stockholm Convention in 1960)by its (then) seven
Member States for the promotion of free trade and economic integration between its members. The
organization operates in parallel with the European Union (EU), and all four member states participate in
the European Single Market and are part of the Schengen Area. However, they are not a party to the
European Union Customs Union.

Q 11.A
• Recently, the Noida police arrested YouTuber for procuring and selling snake venom for recreational
purposes.
• Snake venom is a highly toxic saliva containing zootoxins that facilitates the immobilization and
digestion of prey. This also provides defense against various predators. Hence, statement 1 is correct.
o This also provides defense against threats. Snake venom is injected by unique fangs during a bite,
whereas some species are also able to spit venom.
• The glands that secrete zootoxins are a modification of the parotid salivary glands found in other
vertebrates and are usually located on each side of the head (and not inside the mouth), below and
behind the eye, and enclosed in a muscular sheath. The venom is stored in large glands called alveoli
which is stored before being conveyed by a duct to the base of channeled or tubular fangs through which
it's ejected. Hence statement 2 is not correct.
• This snake venom has been used as a remedy for arthralgia and arthritis in Ayurveda. At times, people
use it as a drug in a diluted form as it can bind on nicotinic acetylcholine receptors
(nAChRs) causing euphoric or rewarding experiences. Such acts, not only cause the abuse of the
venemous snake, but its consumption can lead to health risks, including drowsiness, euphoria, blurred
vision, and even death.
• Snakes are broadly classified into two groups: venemous and non-venemous.
o Venomous snakes are capable of producing venom, which is typically delivered by injection
using fangs. E.g., King Cobra.
o Members of the garter snake genus are generally considered non-venomous. E.g., Boa, Python,
etc. Hence, statement 3 is not correct.

Q 12.B
• The Liquidity management operations of the Reserve Bank are aimed primarily at maintaining an
appropriate level of liquidity in the banking system by using the fixed/ variable rate repo/reverse repo
under liquidity adjustment facility (LAF), the overnight marginal standing facility (MSF) etc for the
efficient transmission of monetary policy and also meeting the need for durable liquidity in the economy
to facilitate the growth of GDP.
• Repurchase Agreement (Repo): Through repo under LAF, the Reserve Bank of India lends funds to
banks and Primary Dealers (PDs) which have both current account and SGL account with the Reserve
Bank of India. Thus repo under LAF is used by RBI for the injection of liquidity. Today, the current
repo rate stands at 6.50% as per the recent update of 8th February 2024.
5 www.visionias.in ©Vision IAS
https://upscpdf.com/ https://upscpdf.com/
https://t.me/visioniastestseries2024
https://upscpdf.com/ https://upscpdf.com/

.
• Reverse Repo: This is exactly the opposite of the repo transaction and is used for the absorption of
liquidity. The reverse repo rate is fixed below the repo rate. The Reverse Repo facility is available to
banks and PDs. The current reverse repo rate remains unchanged at 3.35%.
• Marginal Standing Facility (MSF): The Reserve Bank, in 2011, introduced the Marginal Standing
Facility (MSF) for banks and primary dealers to reduce the volatility in the inter-bank call money market.
The idea of this standing facility is to enable banks to obtain funds from the Reserve Bank when all other
options have been exhausted. The funds under this facility can be availed at the MSF rate which is fixed
certain basis points above the repo rate, as decided by the Reserve Bank from time to time. Hence the
correct order is Reverse repo<Repo<MSF rate. The current MSF rate is 6.75%.
• Hence option (b) is the correct answer.

Q 13.B
• India’s role in global oil markets is expected to expand substantially over the remainder of the decade,
fuelled by strong growth in its economy, population, and demographics. India is set to maintain its
position as a key exporter of transportation fuels to markets in Asia and the Atlantic Basin. As a relatively
small oil producer, and with limited potential for near-term growth, India’s domestic production
accounted for just 13% of the country’s supply needs. In 2023, domestic oil production averaged
around 700 kb/d. Despite renewed efforts by the government to attract foreign upstream investment,
domestic crude oil production is expected to see continued declines over the medium term. Hence
statement 1 is not correct.
• Mumbai High, formerly Bombay High is the largest oil field in India. It is an offshore oilfield
located in the Arabian Sea, approximately 160km west of the Mumbai coast. The field still has reserves
that can continue to produce for a few more years. It had about 1.659 billion tonnes of total reserves in
place. Hence statement 2 is correct.
• Oil extracted from the wells is crude oil and contains many impurities. It cannot be used directly. It needs
to be refined. There are two types of refineries in India: (a) field-based and (b) market-based. Digboi is an
example of field-based and Barauni is an example of market-based refinery. Hence statement 3 is
correct.
o The Barauni refinery is owned by Indian Oil and is located on the Ganges River in Bihar, India. It was
built in 1964 with the help of Romania and the Soviet Union and is one of the oldest refineries in
India.

Q 14.B
• Pulses are annual leguminous crops yielding between one and 12 grains or seeds of variable size, shape
and colour within a pod, used for both food and feed.
• The total world acreage under pulses is about 93.18 (Mha) with production of 89.82 (Mt) at 964 kg/ha
yields level. India, with >28 Mha pulses cultivation area, is the largest pulse-producing country in the
world. It ranks first in area and production with 31 percent and 28 per cent respectively.
• Pulses account for around 20 percent of the area under food grains and contribute around 7-10 percent of
the total food grains production in the country. Hence statement 1 is correct.
• Pulses are grown in all three seasons. The three crop seasons for the commodity are:
o Kharif: Arhar (Tur), Urd (Blackgram), Moong (Green gram), Lobia (Cowpea), Kulthi (Horsegram)
and Moth;
o Rabi: Gram, Lentil, Pea, Lathyrus and Rajmash;
o Summer: Greengram, Blackgram, and Cowpea.
• Though pulses are grown in both Kharif and Rabi seasons, Rabi pulses contribute more than 60
percent of the total production. Hence statement 2 is not correct.
• Under individual crop category gram with 45 percent production share to total pulses is the highest
contributor followed by tur (17 percent), urd (11%), mung (10%) and Lentil (6%). Hence statement 3 is
correct.
• India is the world's largest importer of pulses (and not the largest exporter), accounting for 14% of
global production. India is also the world's largest producer (25%) and consumer (27%) of pulses. India
imports nearly 5 million tons of pulses every year, with annual imports reaching $2.1 billion. Some of the
countries from which India has imported pulses in the past are Australia and Canada. Hence
statement 4 is not correct.

6 www.visionias.in ©Vision IAS


https://upscpdf.com/ https://upscpdf.com/
https://t.me/visioniastestseries2024
https://upscpdf.com/ https://upscpdf.com/

.
Q 15.D
• The Balance of payment account of India is a systematic statement of all economic transactions between
the residents of India and the residents of the rest of the world in an accounting period (say one year).
• The BoP as a classification format classifies the BoP account into two:
o Current account transactions that involve exports and imports of goods and services (services are
incorporated under Invisibles).
o Capital account transactions that involve the flow of investable money to and from India.
• Components of Current Account:
o The Current Account has two components – exports and imports of goods and exports and imports of
Invisibles (includes services). Hence the current account has two subcomponents: Merchandise trade
account ( for exports and imports of goods) and
o The Invisible account ( for services, remittances and income): The merchandise trade account
gives the money value of India’s exports and imports of goods. When we often mention exports and
imports, it is about the merchandise account.
o Invisible accounts indicate India’s Service exports and imports (software exports, tourism
revenues, etc., various service imports)Remittances (private remittances from abroad and
payment to foreign countries)Income (income earned by MNCs from their investment in India).
• Under the Capital Account, capital inflows are classified by instrument (debt or equity) and
maturity (short or long-term).
• The main components of the capital account include foreign investment, loans, and banking capital.
o Foreign investment, comprising Foreign Direct Investment (FDI) and
o Portfolio Investment consisting of Foreign Institutional Investors (FIIs) investment, American
Depository Receipts/Global Depository Receipts (ADRs/GDRs) represents non-debt liabilities,
o while loans (external assistance, external commercial borrowings and trade credit) and banking
capital, including non-resident Indian (NRI) deposits are debt liabilities. Hence option (d) is the
correct answer.

Q 16.C
• Evaporation:
o Evaporation is known as the process by which water converts from a liquid state to a gaseous or vapor
state. In atmospheric terms, it is known as the motion of water molecules from various water bodies to
the surrounding unsaturated air.
o Evaporation is the major pathway of the water cycle through which water moves from the liquid state
back into the atmosphere as water vapor.
o It is estimated that about six hundred calories (in terms of latent heat) are required to convert one
gram of water to vapor state. Nearly 90% of the moisture in the atmosphere is due to the evaporation
of water from the water bodies and the remaining 10% is added by the process of plant transpiration.
• Major environmental factors that affect the rate of evaporation in the atmosphere are:
o Amount of water available:
▪ The availability of water is directly proportional to the rate of evaporation. This is the reason the
rate of evaporation is higher over the oceans.
o Temperature:
▪ The higher the temperature of the air or the evaporating surface, the greater the evaporation rate.
As known, warmer air has more water-holding capacity than cold air and can thus carry more
moisture.
o Humidity:
▪ The higher the humidity, the lower the rate of evaporation. The water-holding capacity of air
at any particular temperature is limited depending on the moisture it is already carrying. As a
result, drier air is capable of evaporating more water than moist water. Hence option 2 is correct.
o Wind speed:
▪ Wind speed is directly proportional to the rate of evaporation i.e. greater the wind speed, the
greater the rate of evaporation. • Area of water surface: The larger the area of exposed water
surface higher the rate of evaporation. Hence option 3 is not correct.
o Air Pressure:
▪ If the pressure of air is low over the water surface, it leads to a higher rate of
evaporation. Hence option 4 is correct.
o Salinity of Water:
▪ The higher the salinity of water, the lower the rate of evaporation. Hence option 1 is correct.

7 www.visionias.in ©Vision IAS


https://upscpdf.com/ https://upscpdf.com/
https://t.me/visioniastestseries2024
https://upscpdf.com/ https://upscpdf.com/

.
Q 17.B
• India is striving to establish a reliable lithium supply to bolster its growing electric vehicle (EV) industry
with its recent Rs 200 crore investment in lithium exploration in Argentina by the Ministry of Mines.
Despite these efforts, industry experts caution that the nation's lithium mining and processing sector is
essentially starting from scratch with a limited immediate impact expected from the latest developments.
• Currently, India imports 100% of its lithium, with a significant portion (over 95%) sourced from
China and Hong Kong.
o Argentina is the part of “Lithium Triangle” along with Chile and Bolivia with more than half of the
world’s total lithium resources and has the distinction of having the 2nd largest lithium resources, 3rd
largest lithium reserves, and the 4th largest production in the world. Hence statement 1 is correct
and statement 2 is not correct.
• Ministry of Mines, Government of India has achieved a significant milestone with the signing of an
agreement between Khanij Bidesh India Limited (KABIL) and the state-owned enterprise of Catamarca
province of Argentina.
• This is the first ever lithium exploration and mining project by a Government Company of
India. KABIL (Khanij Bidesh India Ltd.) will start the exploration and development of 5 lithium brine
blocks located in the Catamarca province of Argentina. Hence statement 3 is correct.

Q 18.A
• A key indicator of an economy’s long-run debt sustainability is the differential between interest paid on
government debt and the economy’s nominal growth rate which is known as Interest Rate Growth
Differential (IRGD). Hence statement 1 is correct.
• According to the Economic Survey 2020-21, with the Indian context of potential high growth, the interest
rate on debt paid by the Indian government has been less than India’s growth rate.
• The Survey notes that the phenomenon of a negative IRGD in India, unlike advanced economies, is not
due to lower interest rates but much higher growth rates. Hence statement 2 is not correct.

Q 19.B
• The Constitution deals with the citizenship from Articles 5 to 11 under part II. However, it contains
neither the permanent nor any elaborate provisions in this regard. It does not deal with the problem if
acquisition and loss of citizenship subsequent to its commencement. It empowers the Parliament to enact a
law to provide for such matters and any matter relating to citizenship. Accordingly, the Parliament has
enacted the Citizenship Act, 1955.
• The Citizenship Act of 1955 prescribes five ways of acquiring citizenship, viz, birth, descent, registration,
naturalisation and incorporation of territory.
• The children of foreign diplomats posted in India and enemy aliens cannot acquire Indian citizenship by
birth. Hence statement 1 is correct.
• In India, both a citizen by Birth as well as a Naturalized citizen are eligible for the post of
President whereas in USA only a citizen by birth and nit a naturalised citizen is eligible for the post of the
President. Hence statement 2 is correct.
• If any foreign territory becomes a part of India, the Government of India specifies the persons who
among the people of the territory shall be the citizen of India. Such persons become the citizens of
India from the notified date. For example, when Pondicherry became a part of India, the Government of
India issued the Citizenship (Pondicherry) Order, 1962, under the Citizenship Act, 1955. Hence
statement 3 is not correct.

Q 20.A
• Obelisks:
o A new genetic entity dubbed ‘obelisks’ has recently been discovered in abundant numbers inside
bacteria inhabiting the human gut and mouth.
o This never-before-seen biological construct displays unique characteristics unlike other known viruses
or cell types, prompting researchers to classify obelisks as a third fundamental domain that could
help better understand microbial ecology in the human body.
• Description of Obelisks:
o Obelisks are circular bits of genetic material that contain one or two genes and self-organize into a
rod-like shape.
o They were discovered by a team at Stanford University through extensive analysis of genetic material
from bacteria in the human gut

8 www.visionias.in ©Vision IAS


https://upscpdf.com/ https://upscpdf.com/
https://t.me/visioniastestseries2024
https://upscpdf.com/ https://upscpdf.com/

.
• Position in Biological Classification:
o Obelisks fall between viruses and viroids in terms of complexity. Hence option (a) is the correct
answer.
o Like viroids, they have a circular single-stranded RNA genome and lack a protein coat.
o However, similar to viruses, their genomes contain genes that are predicted to code for proteins.
• Genomic Characteristics:
o All obelisks encode a single major protein called obulin, with many also encoding a smaller obulin.
o These proteins bear no resemblance to any other known proteins, and their function remains largely
unknown.
• Host and Replication:
o Obelisks likely rely on microbial host cells, such as bacteria or fungi, for replication.

Q 21.B
Types of majority under India constitution:
• Simple Majority: It means a majority of more than 50% of members are present on a particular day and
voting on any bill or issue. For instance, there are a total of 543 seats in Lok Sabha and only 400 members
are present on the day of voting. So, to pass a bill by a simple majority, a minimum of 201 votes will be
required [(50% of 400) +1].
• Absolute Majority: It means 50% + 1 of the total strength of the house.For instance, in the Lok Sabha, it
will be 273 or more (i.e., 50% of 543+1).
• Effective Majority: It means more than 50% of the effective strength of the house where effective
strength is total strength minus vacancies.
• Special majority according to various Articles of the Indian Constitution are as follows:
o According to Article 249 (when Rajya Sabha legislates in matters of State list) and Article 312 (when
Rajya Sabha creates new All India Services), a special majority means 2/3rd of the members present
and voting.
o For instance, in the case of Lok Sabha, out of 543 only 450 members are present at the time of
voting. Thus, the special majority will be 301 (2/3rd of 450 + 1). Hence, statement 1 is correct.
o According to Article 368, a special majority means 2/3rd of the members present and voting plus the
majority of the total membership of the house (Absolute majority).
o For instance, in the case of Lok Sabha, it will be 301 (2/3rd of the members present and voting) plus
273 (Absolute majority).
• The toughest kind of majority requirement under the constitution is given in Article 61. It is used
for the impeachment of the president. In case of Lok Sabha, It requires 2/3rd majority of the total
membership of the Lok Sabha. It means 2/3rd of 543 seats, i.e., 362. Hence, statement 2 is not
correct.

Q 22.B
• A landslide is the rapid movement of rock, soil and vegetation down the slope under the influence of
gravity. These materials may move downwards by falling, toppling, sliding, spreading or flowing. Such
movements may occur gradually, but sudden sliding can also occur without warning. They often take
place in conjunction with earthquakes, floods and volcanic eruptions. The extent and Intensity of
landslides depend upon several factors- Steepness of the slope, amount of vegetation cover, tectonic
activity, bedding plane of the rocks etc.
• Landslide Vulnerability Zones
o Very High Vulnerability Zone: This zone includes the Himalayas, Andaman and Nicobar, high
rainfall regions with steep slopes in the Western Ghats and Nilgiris, the northeastern region. Hence
statement 1 is correct.
o Moderate to Low Vulnerability Zone: Landslides occur on occasion in locations that receive less
precipitation, such as the Trans-Himalayan areas of Ladakh and Spiti (Himachal Pradesh) and
low precipitation areas in the Aravali, rain shadow areas in the Western and Eastern Ghats and
Deccan plateau. Subsidence and mining-related landslides are most common in Jharkhand, Orissa,
Chhattisgarh, Madhya Pradesh, Maharashtra Andhra Pradesh, Karnataka, Tamil Nadu, Goa and
Kerala. Hence statement 2 is correct.
• According to the Indian Landslide Susceptibility Map (ILSM) created by IIT-Delhi 13.17% of India's
geographical area is susceptible to landslides. Geological Survey of India estimates a slightly lower figure
at 12.6%. 4.75% of the area is categorized as "very highly susceptible".

9 www.visionias.in ©Vision IAS


https://upscpdf.com/ https://upscpdf.com/
https://t.me/visioniastestseries2024
https://upscpdf.com/ https://upscpdf.com/

.
• Regional Variation in Susceptibility:
o Sikkim has the highest percentage of land area (57.6 %) susceptible to landslides. Hence
statement 3 is not correct.
o Arunachal Pradesh has the highest absolute area susceptible to landslides.

Q 23.C
• Cadmium is a non-nutritive metal considered harmful to the environment and humans, affecting mainly
the kidneys and the skeleton. It is also a carcinogen by inhalation. The accumulation of cadmium in the
soil, through the use of fertilizers, can affect crops and ultimately animals and humans. Its known
accumulation in the food chain has the potential to cause chronic diseases of the renal, pulmonary,
cardiovascular and musculoskeletal systems.
• Some of the important sources of cadmium pollution are:
o Phosphate fertilizer manufacture and use: Cadmium can be present as an impurity in phosphate
rock, a common raw material used in the manufacture of phosphate fertilizer.
o Battery waste: Cadmium is commonly used in batteries, particularly in rechargeable nickel-cadmium
(NiCd) batteries. Improper disposal or recycling of these batteries can release cadmium into the
environment, contaminating soil and water sources.
o Cigarette smoke: Tobacco plants have a natural ability to absorb cadmium from soil. When tobacco
leaves are smoked, cadmium is released into the air as part of cigarette smoke.
o Coal combustion: Cadmium is present in coal as a trace element. When coal is burned for energy
production, cadmium can be released into the atmosphere as a component of coal combustion
emissions, contributing to air pollution.
o Hence option (c) is the correct answer.

Q 24.B
• India has the largest reserve of iron ore in Asia. The two main types of ore found in our country are
haematite and magnetite. It has great demand in international market due to its superior quality. The iron
ore mines occur in close proximity to the coal fields in the north- eastern plateau region of the country
which adds to their advantage.
• About 95 per cent of total reserves of iron ore is located in the States of Odisha, Jharkhand,
Chhattisgarh, Karnataka, Goa, Telangana, Andhra Pradesh and Tamil Nadu.
• In Odisha, iron ore occurs in a series of hill ranges in Sundergarh, Mayurbhanj and Jhar. The
important mines are Gurumahisani, Katamati, Sulaipet, Badampahar (Mayurbhaj), Kiruburu
(Kendujhar) and Bonai (Sundergarh). Hence pairs 1 is not correctly matched and pair 2 is correctly
matched.

10 www.visionias.in ©Vision IAS


https://upscpdf.com/ https://upscpdf.com/
https://t.me/visioniastestseries2024
https://upscpdf.com/ https://upscpdf.com/

.
• Jharkhand has some of the oldest iron ore mines and most of the iron and steel plants are located around
them. Most of the important mines such as Noamundi and Gua are located in Poorbi and Pashchimi
Singhbhum districts.
• Chhattisgarh: This belt further extends to Durg, Dantewara and Bailadila. Dalli, and Rajhara in
Durg are the important mines of iron ore in the country. Hence pair 3 is correctly matched.
• In Karnataka, iron ore deposits occur in Sandur -Hospet area of Ballari district, Baba Budan hills and
Kudremukh in Chikkamagaluru district and parts of Shivamogga, Chitradurg and Tumakuru districts.
• The districts of Chandrapur, Bhandara and Ratnagiri in Maharashtra, Karimnagar and Warangal district of
Telangana, Kurnool, Cuddapah and Anantapur districts of Andhra Pradesh, Salem and Nilgiris districts of
Tamil Nadu are other iron mining regions.

Q 25.B
• Private income: Private income consists of factor incomes earned within the domestic territory and
abroad by private enterprises and workers (factor owners in the private sector) and current transfer from
the government and the rest of the world.
o Private income = Income from domestic product accruing to the private sector + Net factor income
from abroad + national debt interest+ current transfers from the government + other current transfers
from the rest of the world (net)
• Personal income: Personal income is defined as the current income of persons or households from all
sources. We have to deduct undistributed profit and corporate tax payable by the enterprise from private
income to arrive at personal income. Hence statement 2 is correct.
o Personal income = private income - saving of private corporate sector (undistributed profit) -
corporation tax
• Personal disposable income: The household cannot spend the entire personal income. The government
takes away a part of it by way of inome tax and other miscellaneous taxes such as education tax, fire tax,
and sanitation tax. These taxes have to be deducted from personal income to arrive at personal disposable
income. Personal disposable income is the income available to persons from all sources to dispose of
as they choose. Hence statement 1 is not correct.

Q 26.A
• The Constitution, under Sixth Schedule (Article 244), contains special provisions for the administration of
tribal areas in the four northeastern states of Assam, Meghalaya, Tripura and Mizoram. These are
constitutional devices to protect the economic interests and cultural identities of the tribal communities.
• Inclusion under this Schedule would allow the creation of Autonomous District and Regional
Councils (ADCs and ARCs) — elected bodies with the power to administrate tribal areas. This
would include the power to make laws on subjects such as forest management, agriculture,
administration of villages and towns, inheritance, marriage, divorce and social customs. Hence,
statement 1 is correct.
• When the British took over then Assam, their imposition of formal laws on the land was resisted
ferociously by tribal populations living in the hills, who had their own customary laws. To avoid
confrontation — as they were only interested in the economic exploitation of the region — the
British divided the hill regions of Assam into “excluded” and “partially excluded” areas through
the Government of India Act, of 1935. In these areas, federal or provincial laws would not apply
until the governor felt they were needed for peace and development.
• When India became independent, this provision was adopted with improvements into the Sixth
Schedule of the Constitution based on recommendations made by a committee under then Assam
premier Gopinath Bordoloi. Currently, there are 10 ADCs under the Sixth Schedule in the North
East, with three each in Assam, Meghalaya and Mizoram, and one in Tripura. Manipur has six
ADCs, but these came into existence in 1971 under an act of Parliament. The Tribal sub-plan
originated in the fifth five-year plan (1974-79). Hence, statement 2 is not correct.
• The ADCs and ARCs may also constitute village councils or courts to decide disputes between parties
from Scheduled Tribes, and appoint officers to oversee the administration of the laws they enact. In cases
where the offenses are punishable with death or more than five years of imprisonment, the Governor of
the state can confer upon the ADCs and ARCs the power to try them under the country’s criminal and
civil laws.
• The Schedule also gives ARCs and ADCs the power to collect land revenue, impose taxes, regulate
money lending and trading, collect royalties from licenses or leases for the extraction of minerals in their
areas, and establish public facilities such as schools, markets, and roads. Hence, these act like miniature

11 www.visionias.in ©Vision IAS


https://upscpdf.com/ https://upscpdf.com/
https://t.me/visioniastestseries2024
https://upscpdf.com/ https://upscpdf.com/

.
states having specific powers and responsibilities in respect of all the three arms of governance:
Legislature, executive and judiciary. Hence statement 4 is correct.
• The Sixth Schedule, however, discriminates against the non-tribal residents in various ways and
infringes upon their fundamental rights, like the right to equality before law (Article 14), the right
against discrimination (Article 15), and the right to settle anywhere in India (Article 19). Hence,
statement 3 is correct.

Q 27.A
• Sea Buckthorn is a shrub that produces an orange-yellow coloured edible berry that is sour in taste but
rich in vitamins, especially Vitamin C. Ladakh Sea Buckthorn is a vital crop in Union Territory Ladakh,
with 90% of its production originating from the region.
• It is also popularly known as 'Wonder Plant', “Ladakh Gold”, “Golden Bush” or “Gold Mine”.
• In India, it is found above the tree line in the Himalayan region, generally in dry areas such as the cold
deserts of Ladakh and Spiti. It is a soil-binding plant that prevents soil erosion, checks siltation in rivers
and helps preserve floral biodiversity.
• GI products from Ladakh: Ladakh Pashmina, Apricot (Raktse Carpo species) and Ladakhi wood
carvings.
• Other GI-recognised products:
o Adi Kekir is a Ginger variety from Arunachal Pradesh.
o Ramban Anardana, locally referred to as Dhruni, is an important fruit tree growing wild in hilly tracts
and forests of J&K.
• Hence option (a) is the correct answer.

Q 28.B
• Effective legislative control over the expenditure of the government requires the Parliament to satisfy
itself that the appropriations have been utilized economically for the approved purposes within the
framework of the grants. It should also undertake a detailed examination of the annual budget estimates of
the government to suggest possible economies in the implementation of plans and programs embodied
therein. Both these functions are of pivotal importance in making the parliamentary control over
governmental expenditure comprehensive. The legislature as such has neither the energy nor the time to
perform these functions. It; therefore, constituted three committees, composed of members belonging to it.
to devote themselves to these functions. These three committees are:
• Public Accounts Committee:
o This committee was set up first in 1921 under the provisions of the Government of India Act of 1919
and has since been in existence.
o At present, it consists of 22 members (15 from the Lok Sabha and 7 from the Rajya Sabha). The
members are elected by the
o Parliament every year from amongst its members according to the principle of proportional
representation by means of the single transferable vote.
o The function of the committee is to examine the annual audit reports of the Comptroller and
Auditor General of India (CAG), which are laid before the Parliament by the President. The
CAG submits three audit reports to the President, namely, an audit report on appropriation
accounts, an audit report on finance accounts and an audit report on public undertakings.
Hence, statement 1 is correct.
• Estimates Committee:
o The origin of this committee can be traced to the standing financial committee set up in 1921.
The first Estimates Committee in the post-independence era was constituted in 1950 on the
recommendation of John Mathai, the then finance minister. Originally, it had 25 members but
in 1956 its membership was raised to 30. All the thirty members are from Lok Sabha only. The
Rajya Sabha has no representation in this committee. Hence, statement 2 is correct.
o These members are elected by the Lok Sabha every year from amongst its own members, according to
the principles of proportional representation by means of a single transferable vote.
o The function of the committee is to examine the estimates included in the budget and suggest
‘economies’ in public expenditure. Hence, it has been described as a ‘continuous economy
committee’.
• Committee on Public Undertakings:
o Till April 1964 the affairs of Public Enterprises in India used to be looked after by two Committees:
namely the Estimates Committee and Public Accounts Committee. But in view of huge investments

12 www.visionias.in ©Vision IAS


https://upscpdf.com/ https://upscpdf.com/
https://t.me/visioniastestseries2024
https://upscpdf.com/ https://upscpdf.com/

.
and manifold increase in the activities of public enterprises, it was felt that there should be a separate
agency which should look into the working of public enterprises in detail and report to the Parliament.
o In 1964, on the recommendation of the Krishna Menon Committee, a separate committee on
Public Undertakings was constituted. Hence, statement 3 is not correct.
o This committee, which started functioning on May 1, 1964, took over the work relating to
autonomous Public Enterprises from the other two Committees viz the Estimates Committee and the
Public Accounts Committee.

Q 29.B
• An Asset Management Company (AMC) is a firm that invests the funds pooled from individual
investors in securities with the objective of optimal return for investors in exchange for a fee. AMC
maintains the diversity of its portfolio by investing in both high-risk and low-risk securities such as stock,
debt, real- estate, shares, bonds, pension funds, etc. For example, a debt fund invests in bonds and risk-
free Government bonds to maintain the minimum risk. On the other side, an equity-oriented fund will
invest in shares and stocks with high risk and high return. Hence statement 1 is correct.
• How does an Asset Management Company manage the Funds?
o Research and Analysis: To build a portfolio for an investor, the fund manager conducts extensive
research on market trends, macroeconomic & microeconomic factors, and political aspects. Based on
this research, the appropriate securities are selected that will outperform the return expectations of the
investors.
o Asset Allocation: Based on market research and the investor's financial objectives, the asset manager
allocates the funds to different assets. For example, a debt-oriented fund might invest only 20% in
equity-oriented funds to keep risk levels low, while an equity-oriented fund might invest more than
70% in equity and the rest in debt. A balanced fund might end up with 60% in equity and 40% in debt
to balance return and risk.
▪ Having said that, AMCs have to follow the asset allocation approved by the Securities
Exchange Board of India (SEBI) for every scheme as per its respective category. Hence
statement 2 is correct.
o Performance Review: AMCs are accountable to their investors for the investments they make. To
avoid a negative reputation and criticism from investors, it is crucial to regularly evaluate the fund's
performance, considering factors such as the net asset value (NAV) and fund returns. This helps
AMCs maintain a good reputation and keep their investors satisfied.
• Governing Bodies for an AMC
o Under the oversight of a board of trustees, Asset Management Companies (AMCs) operate in India
under the guidance of two major regulatory bodies:
▪ The Securities and Exchange Board of India (SEBI)
▪ The Association of Mutual Funds in India (AMFI)
▪ Hence statement 3 is not correct.
Q 30.C
• The Youth20 (Y20) is the official youth engagement group for the G20 (Group of 20). The first ever
Y20 Summit held in India on the sidelines of the G20 Summit.
• It will focus on themes of the future of work; climate change and disaster risk reduction; peacebuilding
and reconciliation; and youth in democracy.
• It will focus on global youth leadership and partnership and provides a platform to the youth to express
their vision and ideas on the G20 priorities.
• Hence option (c) is the correct answer
About G20:
• The G20 was founded in 1999 after the Asian financial crisis as a forum for the Finance Ministers and
Central Bank Governors to discuss global economic and financial issues.
• It was upgraded to the level of Heads of State/Government in the wake of the global economic and
financial crisis of 2007, and, in 2009, was designated the “premier forum for international economic
cooperation”.
• It comprises 19 countries (Argentina, Australia, Brazil, Canada, China, France, Germany, India,
Indonesia, Italy, Japan, Republic of Korea, Mexico, Russia, Saudi Arabia, South Africa, Turkey, United
Kingdom and United States) ,the European Union and the African Union.
• The G20 Summit is held annually, under the leadership of a rotating Presidency.
• The G20 members represent around 85% of the global GDP, over 75% of the global trade, and about two-
thirds of the world population.

13 www.visionias.in ©Vision IAS


https://upscpdf.com/ https://upscpdf.com/
https://t.me/visioniastestseries2024
https://upscpdf.com/ https://upscpdf.com/

.
Q 31.A
• Popular sovereignty is government based on consent of the people. The government's source of authority
is the people, and its power is not legitimate if it disregards the will of the people. Government established
by free choice of the people is expected to serve the people, who have sovereignty, or supreme power.
• As the written Constitution possess the validity of a statute emanating from the sovereign people, which is
superior to the present elected members of the legislative. In conclusion, the principle of popular
sovereignty was woven into the Constitution and the people were sovereign in making the constitution.
• The Preamble in its present form reads:
“We, THE PEOPLE OF INDIA, having solemnly resolved to constitute
India into a SOVEREIGN SOCIALIST SECULAR DEMOCRATIC REPUBLIC
and to secure to all its citizens:
JUSTICE, Social, Economic and Political;
LIBERTY of thought, expression, belief, faith and worship;
EQUALITY of status and of opportunity; and to promote among
them all; FRATERNITY assuring the dignity of the individual and
the unity and integrity of the Nation;
IN OUR CONSTITUENT ASSEMBLY this twenty-sixth day of
November, 1949, do HEREBY ADOPT, ENACT AND GIVE TO
OURSELVES THIS CONSTITUTION”
• The preamble, hence, places the sovereignty in the people of India embodying the concept of
popular sovereignty. Hence, option (a) is the correct answer.

Q 32.B
• Fiscal drag is an economic term whereby inflation or income growth moves taxpayers into higher tax
brackets. This in effect increases government tax revenue without actually increasing tax rates. The
increase in taxes reduces aggregate demand and consumer spending from taxpayers as a larger share of
their income now goes to taxes, which leads to deflationary policies, or drag, on the economy.
• Fiscal drag is essentially a slowing in the growth of the economy caused by a lack of spending as
increased taxation slows the demand for goods and services. When an economy is rapidly expanding,
inflation results in higher income and therefore individuals moving into higher tax brackets and paying
more of their income in taxes. This is particularly the case in economies with progressive taxes, or tax
brackets, which stipulate that the higher income an individual makes the higher the tax they pay, and thus
they move into a higher tax bracket.
• Moving into a higher tax bracket and paying a larger portion of income in taxes results in an
eventual slowing of the economy as there is now less income available for discretionary spending.
Hence option (b) is the correct answer.

Q 33.B
• The Prevention of Money Laundering Act (PMLA) is an Indian law that criminalizes the act of
disguising the origin of illegally gained money. The law aims to prevent and control money laundering
activities. It ensures the legality of financial transactions and promotes the integrity of financial systems.
• The Act expressly states that if a person is found guilty of money laundering in India, he will be sentenced
to rigorous imprisonment ranging from 3 to 7 years, and if the proceeds of guilt are related to any of the
offences listed in paragraph 2 of Part A of the Schedule (Offenses under the Narcotic Drugs and
Psychotropic Substance Act, 1985), the sentence will be increased to 10 years.
• Appellate Tribunal: It is the body appointed by the Government of India to hear appeals from decisions
of the adjudicating authority or any other authority established under the Act. It is worth noting that
tribunal decisions can be appealed to the High Court (for that jurisdiction) and then to the Supreme
Court.
• The PMLA schedule has expanded from six to 30 offences since 2002. In April 2023, the Ministry of
Finance expanded the scope of PMLA by adding the activities related to Virtual Digital Assets (VDA)
and Crypto currency under the Act. By this move, VDA entities could end up dealing directly with
enforcement agencies, like ED (Enforcement Director).
• In July 2023, Goods and Services Tax Network (GSTN) was included in the PMLA, allowing
information sharing between GSTN, ED, and other agencies, modifying Section 66 provisions.
• The Supreme Court recently refused to review a 2023 judgment which held that the Prevention of
Money Laundering Act (PMLA) cannot be mechanically invoked merely by citing a standalone
criminal conspiracy charge under Section 120B of the Indian Penal Code (IPC) [Alliance University

14 www.visionias.in ©Vision IAS


https://upscpdf.com/ https://upscpdf.com/
https://t.me/visioniastestseries2024
https://upscpdf.com/ https://upscpdf.com/

.
and anr vs Pavana Dibbur and anr].The Court had explained that for a PMLA case to be initiated on
allegations of a criminal conspiracy, the conspiracy must be linked with a money-laundering offence
already included in Parts A, B or C of the Schedule to PMLA. Hence option (b) is not correct.
• The PMLA’s provisions on admissibility of statements made by the accused as evidence, and the stringent
bail provisions have been the most contentious. It is the only Act in the country in which a statement
recorded before an investigating officer is admissible in court as evidence. Other laws that
contained such provisions, such as TADA and POTA, have long been repealed.
• Hence option (b) is the correct answer.

Q 34.B
• Articles 239 to 241 in Part VII of the Constitution deal with the union territories. Even though all the
union territories belong to one category, there is no uniformity in their administrative system.
• Every union territory is administered by the President acting through an administrator appointed by him.
An administrator of a union territory is an agent of the President and not head of state like a
governor. Advisory Committees of Union Territories
• Under the Government of India (Allocation of Business) Rules 1961, Ministry of Home Affairs is the
nodal ministry for all matters of Union Territories relating to legislation, finance and budget, services and
appointment of Lt. Governors and Administrators.
• All the five UTs without legislature (Andaman and Nicobar Islands, Chandigarh, Daman and diu,
Dadra and Nagar Haveli and Lakshadweep) have the forum of Home Minister’s Advisory
Committee (HMAC)/ Administrator’s Advisory Committee (AAC). Hence statement 1 is correct.
• While HMAC is chaired by the Union Home Minister, AAC is chaired by the Administrator of the
concerned UTs. Hence statement 2 is not correct.
• Members of Parliament and elected members from the local bodies e.g. District Panchayats and Municipal
Council of the respective UTs are members of these committees among others.
• The Committee discusses the general issues relating to social and economic development of the
UTs. Hence statement 3 is correct.

Q 35.D
• The Constitution of India (Article 148) provides for an independent office of the Comptroller and Auditor
General of India (CAG). He is the head of the Indian Audit and Accounts Department. He is the
guardian of the public purse and controls the entire financial system of the country at both levels–
the Centre and the State. Hence, option (d) is the correct answer.
• His duty is to uphold the Constitution of India and the laws of Parliament in the field of financial
administration. This is the reason why Dr. B.R. Ambedkar said that the CAG shall be the most important
Officer under the Constitution of India. He is one of the bulwarks of the democratic system of government
in India; the others being the Supreme Court, the Election Commission and the Union Public Service
Commission
Q 36.D
• Central Ground Water Authority has been constituted under Section 3 (3) of the Environment
(Protection) Act, 1986.
• Purpose: regulation and control of groundwater development and management in the country.
• Functions:
o The Authority is engaged in various activities related to the regulation of groundwater
development to ensure its long-term sustainability.
o It issues the necessary regulatory directions for the purpose.
o It exercises powers under the Act for the appointment of officers.
• India is the largest consumer of Groundwater in the world, which has caused the rapid depletion of
the water tables and aquifers around the country. One such recent example is that of Bengaluru.
• The need of the hour is to utilize water resources judiciously in order to ensure a sustainable future! After
all, Jal hai to Kal hai.
• Hence option (d) is the correct answer.

Q 37.A
• Animal Welfare Board of India (AWBI):
o It is a statutory body.
o It is an advisory body advising the Government of India on animal welfare laws, and promotes animal
welfare in the country of India.
15 www.visionias.in ©Vision IAS
https://upscpdf.com/ https://upscpdf.com/
https://t.me/visioniastestseries2024
https://upscpdf.com/ https://upscpdf.com/

.
o The Animal Welfare Board of India was established in 1962 under Section 4 of The Prevention of
Cruelty to Animals Act, 1960.
o The Board consists of 28 Members, who serve for a period of 3 years.
o It works to ensure that animal welfare laws in the country are followed and provides grants to Animal
Welfare Organisations.
o It takes care of the welfare of domesticated and wild animals in captivity. Hence statement 2 is
correct.
• The Board was initially within the jurisdiction of the Government of India’s Ministry of Food and
Agriculture. Now it is under the administrative control of the Ministry of Fisheries, Animal Husbandry,
and Dairying. Hence statement 1 is not correct.
• It frames a range of rules on how animals ought to be humanely treated everywhere. It has also frequently
litigated to have stricter laws to ensure animals were not unduly harassed or tortured. It's HQ shifted
to Ballabhgarh in Faridabad District of Haryana from Chennai, Tamil Nadu.
• Regulation of Zoos in India:
o Zoos in India are governed by the Wild Life (Protection) Act of 1972 and the National Zoo Policy
of 1992.
o The Wild Life (Protection) Act was amended in 1991, adding a new chapter dealing with zoos to the
Act and allowing the Central Government to establish the Central Zoo Authority to oversee the
functioning and development of zoos in the country. Hence statement 3 is not correct.

Q 38.C
• Normally, respiration is mistaken for breathing - this is not true. Respiration is a chemical reaction. It
occurs in every cell in your body.
• In aerobic respiration, glucose (a type of sugar that you get from food) reacts with oxygen to produce
energy. The energy is needed for growth, repair, and movement. Water and carbon dioxide are bi-products
of respiration - they need to be excreted. This kind of respiration takes place in the mitochondria and that
is why it is called the powerhouse of the cell.
• While most aerobic respiration (with oxygen) takes place in the cell's mitochondria, anaerobic respiration
(without oxygen) takes place within the cell's cytoplasm. Hence option (c) is the correct answer.

Q 39.B
• In simple words, ferromagnetism is a physical phenomenon in which some materials like iron strongly
attract each other. A slightly more technically, ferromagnetism is a property of certain materials (such as
iron) that results in a significant, observable magnetic permeability.
• A ferrimagnetic material is a material that has populations of atoms with opposing magnetic moments, as
in antiferromagnetism, but these moments are unequal in magnitude, so a spontaneous magnetization
remains.
• Some examples of ferromagnetic materials apart from iron are cobalt, nickel, neodymium magnet, and
chromium dioxide. Hence options 1 and 4 are correct.
• Many common metals such as aluminum, copper, brass, gold, silver, titanium, tungsten, and lead are not
ferromagnetic. They cannot be made into magnets and will not be attracted to magnetic fields. Hence
options 2 and 3 are not correct.

Q 40.D
• Salt is essential to the health of humans and other animals, and it is one of the five basic taste sensations.
• Table Salt: The Table salt that we normally consume is a refined salt containing about 97 to 99 percent
sodium chloride. The table salt is white in colour and fortified table salt containes additives like potassium
iodide to prevent iodine deficiency which may cause goitre.
• Rock Salt: Halite, commonly known as rock salt, is a type of salt, the mineral (natural) form of sodium
chloride (NaCl). Halite forms isometric crystals. The mineral is typically colorless or white, but may also
be light blue, dark blue, purple, pink, red, orange, yellow or gray depending on inclusion of other
materials, impurities, and structural or isotopic abnormalities in the crystals.
• Pink Salt: Himalayan salt is rock salt (halite) mined from the Punjab region of Pakistan. The salt, which
often has a pinkish tint due to trace minerals, is primarily used as a food additive to replace refined table
salt but is also used for cooking and food presentation, decorative lamps, and spa treatments. The pink salt
contains nearly 98 percent sodium chloride. Hence statement 1 is not correct.

16 www.visionias.in ©Vision IAS


https://upscpdf.com/ https://upscpdf.com/
https://t.me/visioniastestseries2024
https://upscpdf.com/ https://upscpdf.com/

.
• The Himalayan pink salt contains impurities and that is why it has the distinctive pink hue. Himalayan salt
is used to flavor food. Due mainly to marketing costs, pink Himalayan salt is up to 20 times more
expensive than table salt or sea salt. Hence statement 3 is not correct.
• Black Salt: Kala namak or black salt is a kiln-fired rock salt with a sulphurous, pungent smell used in the
Indian subcontinent. Traditionally, the salt was transformed from its relatively colourless raw natural
forms into the dark coloured commercially sold kala namak through a reductive chemical process that
transforms some of the naturally occurring sodium sulfate of the raw salt into pungent hydrogen sulfide
and sodium sulfide. Thus the pungent smell is due to the presence of sulphur. Hence statement 2 is not
correct.

Q 41.B
• In the Kesavananda Bharati case (1973), the Supreme Court overruled its judgement in the Golak
Nath case (1967). It upheld the validity of the 24th Amendment Act (1971) and stated that
Parliament is empowered to abridge or take away any of the Fundamental Rights. At the same time,
it laid down a new doctrine of the ‘basic structure’ (or ‘basic features’) of the Constitution. It ruled that
the constituent power of Parliament under Article 368 does not enable it to alter the ‘basic structure’ of the
Constitution. This means that the Parliament cannot abridge or take away a Fundamental Right that forms
a part of the ‘basic structure’ of the Constitution.
• The 24th Constitutional Amendment Act of 1971 made it obligatory for the President to give his
assent to a constitutional amendment bill.
• The 44th Amendment to the Constitution made the cabinet’s written advice mandatory for
presidential proclamation of national emergency. This means that the emergency can be declared only
on the concurrence of the cabinet and not merely on the advice of the prime minister.
• Fundamental Duties were added to the constitution by the 42nd amendment act after the Swaran
Singh Committee's recommendations. Hence, option (b) is the correct answer.

Q 42.C
• Coastal Regulation Zone (CRZ) was established in 1991 to regulate the coastal areas, including seas,
bays, creeks, rivers, and backwaters that are subject to tides up to 500 meters from the high tide
line (HTL) and the land between the low tide line (LTL) and the high tide line. According to the
Environment Protection Act of 1986, the CRZ has been declared by the Ministry of Environment, Forest,
and Climate Change.
• The Shailesh Nayak Committee was appointed by the Ministry of Environment, Forest, and
Climate Change (MoEFCC) in June 2014 to conduct a thorough examination of the provisions
under the Coastal Regulation Zone (CRZ) 2011 notification. The committee’s report formed the
basis for the subsequent revision of the CRZ notification in 2018. Hence statement 1 is correct.
• The Coastal Regulation Zone Notification (CRZ) 2019 classifies the coastal area into 4 different zones to
manage infrastructure activities and regulate them. These are:
• CRZ-I:
o CRZ-I constitutes ecologically sensitive areas, such as mangroves, corals/coral reefs, dunes,
national parks, marine parks, sanctuaries, reserve forests, wildlife habitats, etc. The coastal
regulation zone areas in CRZ-I are situated between high tide lines and low tide lines. Hence,
Statement 2 is correct
o High tide line - the line of intersection of the land with the water's surface at the maximum height
reached by a rising tide.
o Low tide line - the line on the land where the lowest tide of water touches during spring tide.
• CRZ-II:
o It constitutes the developed areas up to the shoreline, which fall within the existing municipal
limits. The development of unauthorised structures is not allowed in this zone. Hence, Statement
3 is correct.
• CRZ-III:
o Localities, such as rural areas that are relatively undisturbed and do not fall under the above
categories, are included in this zone. Only specific activities related to agriculture or certain public
facilities are permitted under this coastal regulation zone.
• CRZ-IV:
o The zone constitutes water areas from the low tide line up to territorial limits, including areas of the
tidal-influenced water bodies. Activities such as fishing and other allied services are permitted in this
zone.
17 www.visionias.in ©Vision IAS
https://upscpdf.com/ https://upscpdf.com/
https://t.me/visioniastestseries2024
https://upscpdf.com/ https://upscpdf.com/

.
• Demarcation of the Hazard Line in the Coastal Regulation Zone (CRZ) is typically carried out by
the National Centre for Sustainable Coastal Management (NCSCM), an autonomous organization
under the Ministry of Environment, Forest and Climate Change (MoEFCC). The Hazard Line
delineates areas vulnerable to natural hazards such as erosion, sea-level rise, and coastal flooding. Hence,
Statement 4 is not correct.

Q 43.C
• National Savings Certificate (NSC) is a fixed-income post office savings scheme. It is offered by the
government of India. One has to visit the post office to activate this scheme, which comes with a lock-in
period of 5 years where one cannot extend the certificate beyond this tenure. The interest rate in this
scheme remains fixed throughout the tenure.
• The government has promoted the National Savings Certificate as a savings scheme for individuals.
Hence, Hindu Undivided Families (HUFs) and trusts cannot invest in it. Furthermore, even non-
resident Indians (NRI) cannot purchase NSC certificates. The scheme is open only for individual
Indian residents.
• Features & benefits of NSC
o Fixed income: Currently, the scheme is generating a guaranteed return at the rate of 7.7% for
investors. The returns offered by NSC have generally been higher than FDs.
o Tax saver: As a government-backed tax-saving scheme, one can claim up to Rs.1.5 lakh under the
provisions of Section 80C of the Income Tax Act, 1961. While there is no upper limit on the amount
that can be invested in NSC, only investments of up to Rs.1.5 lakh a year. Hence option (c) is the
correct answer.
o Loan collateral: Banks and NBFCs accept NSC as collateral or security for secured loans. To do
this, the concerned postmaster should put a transfer stamp on the certificate and transfer it to the bank.

Q 44.A
• ‘German Plot’ or the ‘Zimmerman Plan’:
o During the First World War, the Jugantar party arranged to import German arms and
ammunition through sympathizers and revolutionaries abroad.
o Jatin Mukherjee asked Rashbehari Bose to take charge of Upper India, aiming to bring about an all-
India insurrection in what has come to be called the ‘German Plot’ or the ‘Zimmerman Plan’.
Hence option (a) is the correct answer.
o The Jugantar party raised funds through a series of dacoities which came to be known as taxicab
dacoities and boat dacoities, to work out the Indo-German conspiracy.
o It was planned that a guerrilla force would be organized to start an uprising in the country, with a
seizure of Fort William and a mutiny by armed forces. Unfortunately for the revolutionaries, the plot
was leaked out by a traitor.
o Police came to know that Bagha Jatin was in Balasore waiting for the delivery of German arms. Jatin
and his associates were located by the police.
o There was a gunfight and as a result, the revolutionaries were either killed or arrested. The German
plot thus failed.
o Jatin Mukherjee was shot and died a hero’s death in Balasore on the Orissa coast in September
1915.

Q 45.D
• In a groundbreaking study, scientists exploring Southern California's Salton Sea, the state's largest
lake, have unearthed a staggering lithium reservoir worth an estimated $540 billion. The 'white
gold' discovery comes as a game-changer, positioning the United States as a potential leader in
lithium production. Lithium is an element that is essential for electric vehicle (EV) batteries and
renewable energy storage systems. Hence, option (d) is the correct answer.
• The Salton Sea is a shallow, landlocked, highly saline body of water in Riverside and Imperial
counties at the southern end of the U.S. state of California. It lies on the San Andreas Fault within the
Salton Trough, which stretches to the Gulf of California in Mexico. About 40 miles north of the
California-Mexico border lies the shrinking, landlocked lake known as the Salton Sea.
• Lithium is a metal which possesses low density, and high-energy-to-weight ratio. It is highly reactive (as
it easily loses its outermost electron), allowing it to readily generate current flow through a battery. It can
hold large amounts of energy in rechargeable batteries for electronics as well as electric vehicles. This
metal is also used in non-rechargeable batteries for medical devices such as heart pacemakers, cardiac
defibrillators, drug pumps, and neurostimulators.
18 www.visionias.in ©Vision IAS
https://upscpdf.com/ https://upscpdf.com/
https://t.me/visioniastestseries2024
https://upscpdf.com/ https://upscpdf.com/

.
• Lithium is also used in non-rechargeable batteries in everyday items such as flashlights and clocks. This
metal is an essential part of daily life, and its use for it will be increasing even more as demand for electric
vehicles has gone up in recent years. Lithium can also be given in the form of a pill to treat bipolar
disorder. It is also being studied for potentially treating Alzheimer’s. Lithium is also the lightest metal in
the world, making it ideal for use in cell phone batteries as well as cars which require multiple batteries.

Q 46.D
• The right to legal aid is a cornerstone of justice and a fundamental human right that guarantees
everyone, regardless of means, equal access to the judicial system. It is essential to a just and
equitable society because it ensures that everyone has access to justice and that it is not merely a
privilege for the rich.
• Several international human rights treaties are the foundation for the right to legal aid. The significance of
offering legal aid to individuals in need is emphasised in the Universal Declaration of Human Rights, the
International Covenant on Civil and Political Rights, and regional treaties like the European Convention
on Human Rights. These documents acknowledge that a fair trial and equal protection under the law may
not be provided to people without sufficient legal counsel and that access to justice is a crucial component
of the rule of law.
• Access to legal aid is crucial in upholding the principles of due process and fair trials. When individuals
have access to competent legal representation, they can present their cases effectively and navigate
complex legal procedures. This, in turn, enhances the integrity of judicial outcomes and prevents wrongful
convictions or judgments based on inadequacies in legal representation. Both Directive Principles of State
Policy and Fundamental Rights in the Indian Constitution play a role in shaping the framework for legal
aid.
• Directive Principles of State Policy (DPSP): Article 39A of the DPSP mandates that the state make sure
the functioning of the judicial system promotes justice on the basis of equal opportunity. It expressly
states that the state must offer free legal aid through appropriate laws, programmes, or any other method
to ensure that no citizen's access to justice is restricted because of their financial situation or another
obstacle.
• Fundamental Rights: Fundamental Rights are enforceable rights that individuals can directly seek
the protection of the courts for, in contrast to DPSPs which are non-justiciable principles. The right
to life and personal freedom is guaranteed by Article 21 of the Indian Constitution, which is a part
of Fundamental Rights. The judiciary has broadened its interpretation of this right over time to
encompass the right to uncompensated legal aid. This means that if a person cannot afford legal
representation and their personal liberty is at stake, they have the right to access free legal aid.
• The Supreme Court of India, recognizing the importance of legal aid in ensuring a fair trial, has
laid down various guidelines to operationalize the right to free legal aid. For instance, in the case of
Hussainara Khatoon v. Home Secretary, State of Bihar (1979), 11 the court emphasized the right to
free legal aid as an essential element of Article 21 (right to life and personal liberty). Hence, option
(d) is the correct answer.

Q 47.C
• Ib River:
o The Ib River is a tributary of the Mahanadi River in central India, which it joins to flow directly into
the Hirakud Reservoir. The Ib originates in hills near Pandrapet at an elevation of 762 metres.
▪ Location: Odisha. Hence pair 1 is correctly matched.
▪ Length: Approximately 175 kilometers
▪ Tributaries: The Ib River is a tributary of the Mahanadi River.
▪ Significance: The Ib River plays a crucial role in the irrigation and agricultural activities of the
Odisha region.
▪ It is also important for local biodiversity and supports various aquatic life forms.
• Falgu River:
o The Phalgu or Falgu, a river that flows past Gaya, India in the Indian state of Bihar, is a sacred river
for Hindus and Buddhists. Lord Vishnu's Temple Vishnupad Mandir is situated on the bank
of Phalgu river also called Niranjana river.
▪ Location: Bihar. Hence pair 2 is correctly matched.
▪ Length: Approximately 290 kilometers
▪ Tributaries: The Falgu River is a tributary of the Ganges River.

19 www.visionias.in ©Vision IAS


https://upscpdf.com/ https://upscpdf.com/
https://t.me/visioniastestseries2024
https://upscpdf.com/ https://upscpdf.com/

.
▪ Significance: The Falgu River is considered sacred in Hinduism and is associated with various
religious sites, including the city of Gaya, a prominent pilgrimage destination. It is also known
for its historical significance and cultural heritage.
• Pavana River:
o The Pavana River is a river in western India. It originates in the Western Ghats and flows through
the state of Maharashtra before joining the Mula River in Pune.
▪ Location: Maharashtra. Hence pair 3 is correctly matched.
▪ Length: Approximately 60 kilometers
▪ Tributaries: The Pavana River is formed by the confluence of the Ramnadi and the
Pavananadi.
▪ Significance: The Pavana River is a key water source for the Pavana Dam, which provides
irrigation water for agricultural activities and drinking water for nearby communities. It is also a
popular destination for recreational activities such as boating and picnicking.

Q 48.A
• The Medical Termination of Pregnancy (MTP) Act, 2021 is an Act of the Parliament of India that
regulates the medical termination of pregnancy. The Act was enacted in 1971 and has been amended
several times since then. It aims to provide access to safe and legal abortion services while ensuring the
health and well-being of women.
• Changes made by the MTP Act, 2021
o Increased the gestational age limit for termination of pregnancy from 20 to 24 weeks.
o Allowed for termination of pregnancy on the grounds of foetal abnormalities.
o Removed the requirement for a second opinion from a medical practitioner beforea termination of
pregnancy can be carried out.
o Simplified the procedure for obtaining a certificate for termination of pregnancy.
• The Act does not specify the marital status of the woman as a factor for determining eligibility for
MTP. Therefore, both married and unmarried women may be eligible for MTP if they meet the
specified conditions. Hence, statement 1 is not correct.
• Under the MTP Act,, the following requirements apply for termination of pregnancy:
o Up to 20 weeks of gestation:
▪ Requires the opinion of one registered medical practitioner. Hence, statement 2 is correct.
o Between 20-24 weeks of gestation:
▪ Requires the opinion of two registered medical practitioners, one of whom must be a gynecologist
or obstetrician.
▪ Requires the opinion of a medical board consisting of a gynecologist or obstetrician, a
pediatrician, and a psychiatrist.
o The Act does not allow for the termination of pregnancy after 24 weeks except in cases of:
▪ When the continuance of the pregnancy poses a risk to the life of the pregnant woman; or
▪ When the fetus has a substantial physical or mental abnormality that is incompatible with life.
▪ Hence, statement 3 is not correct.

Q 49.C
• The book Majm'aul Bahrain Mingling of two Oceans) was written by Prince Dara Shikoh. It is
related to sufism. In this work he has compared the Islamic sufi concepts with Hindu philosophical
outlook. Hence pair 1 is correctly matched.
o He has also written a book called Sakinatul Uliya which is a biographical account of the sufi Miya
Mir and his disciples.
o Dara Shikoh was the eldest son and heir-apparent of the Mughal emperor Shah Jahan
• Padshahnamah was written by Abdul Hamid Lahori. It is very important for reconstruction of the
history of Bengal, because some important events in Bengal have been more or less elaborately discussed
in this book. The first is the expulsion of the Portuguese from Hughli. Hence pair 2 is correctly
matched.
o He also gives a comprehensive account of the Mughal warfare in the northeast frontier of Cooch
Bihar and Assam.
o Another important event that has been discussed by Lahori is the repulsion of an Arakanese attack on
the coastal districts of Bengal.
• The Nuh Sipihr is a masnawi (poetic work) which was written by Amir Khusrau in 1318 and
eulogises Mubarak Shah Khalji. It appears to reflect most perfectly the ideas of Khusrau about India
which he had tried to develop in his earlier works. Hence pair 3 is correctly matched.
20 www.visionias.in ©Vision IAS
https://upscpdf.com/ https://upscpdf.com/
https://t.me/visioniastestseries2024
https://upscpdf.com/ https://upscpdf.com/

.
o This work is divided into nine chapters which correspond to the nine skies or spheres (sipihr) of the
heavens.
o Amir Khusrau, one of the most versatile personalities of medieval India, was born in 1253 in a place
called Patiyali, Uttar Pradesh.
▪ Amir Khusrau is often acknowledged for creating Khayal of north Indian classical music known
as Hindustani. He modified raga Dhrupad and added Persian tunes and beats to it. He created
Qawali on the likes of bhajans
▪ Khusro was a Royal poet under Sultan Aalauddin Khalji.

Q 50.A
• Peer-to-peer (P2P) lending is the practice of lending money to individuals or businesses through online
services that match lenders with borrowers, cutting out the financial institution as the middleman. P2P
lending, also known as "social lending" or "crowd lending", has been around since 2005. Hence
statement 1 is correct.
• The P2P lending websites connect borrowers to lenders. The rates and terms are set by each website, and
it enables the transaction. The P2P lending companies often offer their services online and attempt to
operate with lower overhead and provide their services more cheaply than traditional financial institutions.
This helps lenders earn higher returns compared to savings and investment products offered by banks,
while borrowers can borrow money at lower interest rates, even after the P2P lending company has taken
a fee for providing the match-making platform and credit checking the borrower.
• Peer-to-peer lending is riskier than a savings account or certificate of deposit, but the interest rates
are much higher. This is because those who invest in a peer-to-peer lending site assume most of the risk
that banks or other financial institutions normally assume. Hence statement 2 is not correct.
• In India, peer-to-peer lending is regulated by the Reserve Bank of India (RBI). In 2017, the RBI
published a consultation paper on regulating P2P lending, and the final guidelines were released. Hence
statement 3 is not correct.

Q 51.A
• The International Solar Alliance (ISA) and the United Nations Industrial Development
Organization (UNIDO), with funding of the Government of France, are implementing the project
Structuring of an International Network of Solar Technology and Application Resource Centres
(STAR C)”. Hence statement 1 is not correct.
• The overall objective is to create a strong network of institutional capacities within ISA Member
States to enhance quality infrastructure (QI) for the uptake of solar energy product and service markets,
particularly in least developed countries (LDCs) and small island developing states (SIDS). Hence
statement 2 is correct.
• The first phase of the STAR C project will focus on the Economic Community of West African
States (ECOWAS), the East African Community (EAC) and the Pacific Community (SPC)/Pacific
Island Forum (PIF). Hence statement 3 is not correct.
• To have more impact and to create economies of scale, the STAR C has adopted a regional approach,
which facilitates cross-border harmonisation of solar product, service and qualification standards, pooling
of resources and joint implementation on national level.
o In a first step, STAR C will assist the three economic communities in strengthening the regional solar
QI frameworks and management systems in partnership with the national standardisation bodies.
• Past quality infrastructure interventions of UNIDO have proven the added value and benefit of regional
approaches. In this context, UNIDO has supported several economic communities, including ECOWAS,
EAC and SPC, in the establishment of quality infrastructure policies and regional energy centres under the
Global Network of Regional Sustainable Energy Centres (GN-SEC) program.
o The STAR C is part of the south-south and triangular activities of the centres under the GN-SEC
platform. The STAR C implementation will benefit for the established institutional infrastructure and
lessons learned of these interventions.

Q 52.A
• Puppetry throughout the ages has held an important place in traditional entertainment. Almost all types of
puppets are found in India including- String Puppets, Shadow Puppets, Rod Puppets, and Glove Puppets.
• The string puppets of Odisha are known as Kundhei. Made of light wood, the Odisha puppets have no
legs but wear long flowing skirts. They have more joints and are, therefore, more versatile, articulate, and

21 www.visionias.in ©Vision IAS


https://upscpdf.com/ https://upscpdf.com/
https://t.me/visioniastestseries2024
https://upscpdf.com/ https://upscpdf.com/

.
easy to manipulate. Other types of string puppetry are Kathputli from Rajasthan, Gombeyatta from
Karnataka, and Bommalattam from Tamil Nadu. Hence, statement 1 is not correct.
• Rod puppets are an extension of glove puppets, but are often much larger and supported and manipulated
by rods from below. This form of puppetry is found mostly in West Bengal and Orissa. Examples
include- Putul Nautch from West Bengal and Yampuri from Bihar. Hence, statement 2 is correct.
• India has the richest variety of types and styles of shadow puppets. Shadow puppets are flat figures. They
are cut out of leather and are treated to make them translucent. Shadow puppets are pressed against the
screen with a strong source of light behind them. This tradition of shadow puppets survives in Orissa.
Kerala, Andhra Pradesh, Karnataka, Maharashtra and Tamil Nadu. Other examples of shadow puppetry
include- Togalu Gombeyatta from Karnataka, Tholu Bommalata from Andhra Pradesh, and
Ravanachhaya from Odisha. Hence, statement 3 is not correct.
• Glove puppets are also known as sleeve, hand, or palm puppets. The head is made of either papier mache,
cloth, or wood, with two hands emerging from just below the neck. The rest of the figure consists of a
long flowing skirt. These puppets are like limp dolls, in the hands of an able puppeteer, and are capable of
producing a wide range of movements. Example include Pavakoothu from Kerala.

Q 53.A
• ECBs are commercial loans raised by eligible resident entities from recognized non-resident entities and
should conform to parameters, such as minimum maturity, permitted and non-permitted end-uses,
maximum all-in-cost ceiling, etc.
• Eligible Borrowers:
o All entities eligible to receive FDI are eligible for ECBs
o The following entities are also eligible to raise ECB - Port Trusts; Hence option 2 is correct.
o Units in SEZ;- SIDBI; and - EXIM Bank of India. Hence option 1 is correct.
o All entities eligible to raise FCY ECB - INR denominated ECB
o Registered entities engaged in micro-finance activities - Registered Not for Profit companies-
Registered societies/trusts/cooperatives- Non-Government Organisations# LLPs are not eligible
borrowers as not eligible to receive FDI (LLP eligible to receive Foreign Investment under
FEMA). Hence option 3 is not correct.
o Recognition of Lenders: The lender should be residents of FATF or IOSCO-compliant country,
including on transfer of ECB. However,
• Multilateral and Regional Financial Institutions where India is a member country.• Individuals can
only be permitted if they are foreign equity holders OR For subscription to bonds/debentures listed
abroad• Foreign branches/subsidiaries of Indian banks only for FCY ECB (except FCCBs and
FCEBs),• Can participate as arrangers/underwriters/market-makers/traders for Rupee denominated
Bonds issued overseas• Underwriting by foreign branches/subsidiaries of Indian banks for issuances
by Indian banks will not be allowed.
o Minimum Average Maturity Period for ECB will be 3 years.
• Hence option (a) is the correct answer.

Q 54.C
• The Fundamental Rights are guaranteed by the Constitution to all persons without any discrimination.
They uphold the equality of all individuals, the dignity of the individual, the larger public interest and the
unity of the nation.
• The Fundamental Rights are meant to promote the ideal of political democracy. They prevent the
establishment of an authoritarian and despotic rule in the country, and protect the liberties and
freedoms of the people against the invasion by the State. They operate as limitations on the tyranny
of the executive and arbitrary laws of the legislature. In short, they aim at establishing ‘a
government of laws and not of men’. Hence, option (c) is the correct answer.
• The Fundamental Rights are named so because they are guaranteed and protected by the Constitution,
which is the fundamental law of the land. They are ‘fundamental’ also in the sense that they are most
essential for the all-round development (material, intellectual, moral and spiritual) of the individuals.
Q 55.B
• Sickle cell anemia, characterized by a genetic anomaly inherited through autosomal recessive
disorder, comprises a group of conditions affecting hemoglobin—a protein in red blood cells responsible
for oxygen transport.
• Under normal circumstances, red blood cells are disc-shaped and flexible, allowing easy movement
through blood vessels.

22 www.visionias.in ©Vision IAS


https://upscpdf.com/ https://upscpdf.com/
https://t.me/visioniastestseries2024
https://upscpdf.com/ https://upscpdf.com/

.
• However, in sickle cell disease, a genetic mutation transforms these cells into crescent-shaped or
"sickle" forms. To address the underlying genetic mutation causing this condition, CRISPR-based
approaches aim to intervene. Hence option (b) is the correct answer.
• Clustered Regularly Interspaced Short Palindromic Repeats (CRISPR)-Cas9, a genetic code found in
bacteria, results from the activity of previous bacteriophages. Bacteria use the CRISPR sequence to
memorize specific viruses by incorporating virus DNA into their own genome, providing protection
against future attacks.
• CRISPR-associated protein 9 (Cas9) is a bacterial RNA-guided endonuclease activated to produce
enzymes co-evolving with CRISPR. The three key elements of CRISPR-Cas9 work include Guide RNA,
an engineered RNA piece locating the targeted gene; Cas9, acting as "scissors" to snip out undesired
DNA; and DNA, the desired piece inserted after the break. This sophisticated approach holds promise for
correcting genetic mutations and treating conditions like sickle cell anemia.

Q 56.A
• Under the Constitution, the jurisdiction of each state is confined to its own territory. Hence, it is possible
that the acts and records of one state may not be recognized in another state. To remove any such
difficulty, the Constitution contains the “Full Faith and Credit” clause which lays down the following:
o Full faith and credit is to be given throughout the territory of India to public acts, records and
judicial proceedings of the Centre and every state. The expression ‘public acts’ includes both
legislative and executive acts of the government. The expression ‘public record’ includes any
official book, register or record made by a public servant in the discharge of his official duties.
Hence, statement 1 is correct.
o The manner in which and the conditions under which such acts, records and proceedings are to be
proved and their effect determined would be as provided by the laws of Parliament. This means that
the general rule mentioned above is subject to the power of Parliament to lay down the mode of proof
as well as the effect of such acts, records and proceedings of one state in another state.
o Final judgments and orders of civil courts in any part of India are capable of execution
anywhere within India (without the necessity of a fresh suit upon the judgment). The rule
applies only to civil judgments and not to criminal judgments. In other words, it does not
require the courts of a state to enforce the penal laws of another state. Hence, statement 2 is not
correct.

Q 57.B
• In 1917, a delegation of women activists presented a memorandum of demands to Edwin Montagu
and Lord Chelmsford, who were tasked with formulating a self-governance scheme for India. This
was one of the first official call to enfranchise women.
• The Women's Indian Association (WIA) was formed the same year to address the socio-economic
challenges faced by women, becoming the first national body to advocate for female suffrage. In 1918,
WIA and others intensified their advocacy, traveling to Britain to seek support for their
causes. Sarojini Naidu took the women's rights issue to the Congress party and moved resolutions for
women's enfranchisement at Congress sessions in Bijapur and Bombay.
• The first victory came with the Government of India Act 1919, which allowed provincial legislatures
to enfranchise women.
• In 1921, Madras became the first province to grant women the right to vote, followed by Bombay
and the United Provinces.
• The enfranchisement Bill was defeated in the Bengal Legislative Council. Suffragists led by the Bangiya
Nari Samaj organized massive awareness campaigns for four years, leading to the passage of the Bill in
1925. Women leaders did not let up after the initial success.
• In 1929, an All Parties Conference prepared the Nehru Report, a draft Constitution, which called for equal
civic rights for all citizens. However, Britain was not supportive of expanding this right.
• Hence option (b) is the correct answer.

Q 58.A
• The National Games of India is an Olympic-style multi-sport event where athletes from the various
states and Union Territories of India compete for medals.
o Services Sports Control Board (SSCB), the sports team of the Indian Armed Forces, also competes at
the National Games.

23 www.visionias.in ©Vision IAS


https://upscpdf.com/ https://upscpdf.com/
https://t.me/visioniastestseries2024
https://upscpdf.com/ https://upscpdf.com/

.
o The 1st edition was held at Lahore in an undivided Punjab in 1924. Lucknow hosted the first National
Games after independence.
o The first National Games on the lines of the Olympics were held in 1985 in New Delhi. Hence,
statement 1 is not correct.
• The National Games of India are organized under the jurisdiction of the Indian Olympic Association. It
was founded in 1927 with Sir Dorabji Tata as the Founding President. It is the governing body for the
Olympic Movement and the Commonwealth Games in India registered as a Non-Profit organization under
the Societies Registration Act of 1860. Hence, statement 2 is not correct.
• The recent (37th edition) of the National Games was organized in Goa. The upcoming 38th edition
would be organized in Uttarakhand and the 39th edition in Meghalaya. Hence statement 3 is correct.

Q 59.A
• It was launched in 2022 as a Central Sector Scheme.
• The scheme is being undertaken jointly by the Ministry of Housing and Urban Affairs and the
Ministry of Social Justice & Empowerment (MoSJE). Hence statement 1 is not correct.
• Objectives:
o Zero fatalities in sanitation work in India.
o All sanitation work is performed by skilled workers.
o No sanitation workers come in direct contact with human faecal matter.
o Sanitation workers are collectivised into Self Help Groups (SHGs) and are empowered to run
sanitation enterprises. Hence statement 2 is correct
• The components of the scheme include:
o Profiling of Sewer and Septic tank sanitation workers(SSWs) in ULBs through digital tools (~ 1 lakh
SSW to be identified).
o Health Insurance of SSW under PM-JAY.
o Occupational safety training of SSWs and Sanitation Response Units for NAMASTE.
o Capital Subsidy up to Rs. 5.00 lakh for procurement of Sanitation Related Vehicles/
Equipment. Hence statement 3 is not correct.
o Distribution of PPE to Sewer and Septic tank sanitation workers(SSWs).
o Distribution of safety devices to Emergency Response Sanitation Unit (ERSU).
o Information, Education & Communication (IEC) Campaign for awareness of SSW safety and dignity.

Q 60.A
• Ker means austerity in English. This Puja is celebrated 14 days following the end of Kharchi puja. It is the
strictest puja ever performed by the Tripuri people. In this, all the gods of the Tripuri people are
worshipped at a time. During this puja, a particular area is demarcated, this area is bounded by the Ker
symbol. The road to entry or exit is blocked by the Ker symbol. People are not allowed to light fires,
dance, sing or even wear shoes during this time. Hence, pair 1 is not correctly matched.
• Tamladu Festival is one of the ancient and popular festivals of Arunachal Pradesh. This festival is
celebrated in the Lohit district of the state by the Digaru Mishmis tribe. During this festival, one gets to
witness the display of their social, religious, and supernatural beliefs. Hence, during this festival prayers
are offered to nature. Tamladu Festival is celebrated on 15th February and this festival spreads the
message of unity and gaiety. Throughout the festival, Tanggong dance is performed, which is a popular
traditional dance. Hence, pair 2 is not correctly matched.
• Nua Khai festival celebrates the newly harvested food by the farmers. It is celebrated especially in
the Western part of Odisha one day after the Ganesh Chaturthi. The festival traces its origin to the Vedic
period where the sages or Rishis used to talk about Panchyajna. One among them was Pralambana yajna
which means the cutting of new crops and offering them to the mother goddess as followed in the Nuakhai
festival. Hence, pair 3 is correctly matched.

Q 61.A
• Foundation species are organisms that play a crucial role in shaping and maintaining ecosystems that
support other species. They have a disproportionate impact on their environment, often by creating or
modifying habitats that support a wide range of other species. For example, Corals supporting other
species in the ecosystem.
o Foundation species are typically dominant or abundant in their ecosystems and can significantly
influence ecosystem structure, function, and biodiversity.

24 www.visionias.in ©Vision IAS


https://upscpdf.com/ https://upscpdf.com/
https://t.me/visioniastestseries2024
https://upscpdf.com/ https://upscpdf.com/

.
o These species form the backbone of ecosystems, providing essential resources and habitat for other
organisms.
• A beaver building a dam that creates a pond is an example of a foundation species because
o Habitat Creation: Beavers are ecosystem engineers, meaning they significantly alter their
environment. By constructing dams across streams or rivers, they create ponds or wetlands. These
newly created habitats are often rich in resources and provide homes for a diverse array of organisms.
o Biodiversity Support: The pond created by the beaver dam becomes a focal point for biodiversity. It
supports a variety of plants, animals, and microorganisms that may not thrive in the original stream or
river habitat. Fish, amphibians, birds, insects, and aquatic plants are just some examples of the species
that can benefit from the new habitat.
o Resource Availability: The pond created by the beaver dam provides resources such as water, food,
shelter, and breeding sites for a wide range of organisms. For instance, fish find refuge in the deeper
waters, amphibians lay their eggs in shallow areas, and birds nest in the surrounding vegetation.
o Ecosystem Stability: The presence of the beaver and the pond it creates can enhance ecosystem
stability. Ponds act as buffers against floods and droughts, help regulate water flow, and filter
pollutants from the water. Additionally, the complex interactions between species within the pond
ecosystem contribute to its resilience to environmental changes.
o Dominant Influence: Foundation species like beavers have a disproportionate impact on their
ecosystems compared to their abundance. While beavers themselves are relatively small in number,
their engineering activities fundamentally transform landscapes and influence the distribution and
abundance of numerous other species. This makes them essential for the structure and function of the
ecosystem.
• A top predator regulating the population of prey species: While top predators play important roles in
ecosystems by controlling prey populations and maintaining balance, they typically do not create or
modify habitats in the same way that foundation species do. Instead, top predators are more
often considered keystone species, which have a disproportionately large impact on their ecosystems due
to their ecological roles.
• An invasive species outcompeting native vegetation: Invasive species can have significant impacts on
ecosystems by outcompeting native species, disrupting ecological processes, and altering habitats.
However, they are not considered foundation species because their presence tends to reduce biodiversity
and disrupt ecosystem functions rather than enhancing them.
• A decomposer breaking down organic matter in soil: Decomposers, such as bacteria, fungi, and
detritivores, play crucial roles in nutrient cycling by breaking down organic matter. While they are
essential for ecosystem function, decomposers do not typically create or modify habitats to the extent that
foundation species do. They primarily operate within existing habitats, rather than actively shaping them.
• Hence option (a) is the correct answer.

Q 62.B
• Recent context : Department of Fisheries, Ministry of Fisheries, Animal Husbandry & Dairying, GoI
organized the 19th Working Party on Data Collection and Statistics (WPDCS19) of the Indian Ocean
Tuna Commission (IOTC) from 28th November to 2nd December 2023.
• The Indian Ocean Tuna Commission (IOTC) is an intergovernmental organisation responsible for
the management of tuna and tuna-like species in the Indian Ocean. Hence statement 1 is correct.
o It works to achieve this by promoting cooperation among its Contracting Parties (Members) and
Cooperating Non-Contracting Parties in order to ensure the conservation and appropriate utilisation of
fish stocks and encouraging the sustainable development of fisheries.
• Membership is open to Members and Associate Members of FAO, or by a two-thirds majority of its
Members, any other States that are not Members of FAO, but are Members of the United Nations, or of
any of its Specialized Agencies or of the International Atomic Energy Agency, that are coastal Members
or Associate Members situated wholly or partly within the Area.

25 www.visionias.in ©Vision IAS


https://upscpdf.com/ https://upscpdf.com/
https://t.me/visioniastestseries2024
https://upscpdf.com/ https://upscpdf.com/

.
• Members

• Hence statement 2 is correct.


• Tunas migrate long distances over all the world's oceans and occupy tropical, temperate, and even
some cooler waters. The only two species of relatively limited distribution are the blackfin tuna
(western Atlantic) and the longtail tuna (Indo-Pacific region). Hence statement 3 is not correct.
o Tunas and other large pelagic species, such as billfishes, sharks, and rays, hold immense economic
significance, with tunas alone contributing an estimated annual value of trade of US$41 billion (in
2018).

Q 63.A
• Recently, the United Nations Secretary-General has decided to invoke Article 99 of the UN Charter and
has urged the UN Security Council to act on the humanitarian crisis in Gaza.
• Article 99 of the UN Charter is a special, discretionary power given to the UN Secretary-
General. The article allows the UN Secretary-General to bring to the attention of the UN Security
Council (UNSC) any matter which in his opinion may threaten the maintenance of international
peace and security.
• When Article 99 is invoked, the President of the UNSC is under the obligation to call a meeting of the
Security Council.
• It has only been invoked four times in the past — in the Congo (1960), East Pakistan (1971), Iran
(1979) and Lebanon (1989).
• Article 99 does not provide the UN Secretary General with the means to compel the UNSC to adopt
resolutions.
• Moreover, for a resolution to be adopted, it needs at least nine votes in favor and no vetoes by the five
permanent members – the United States, Russia, China, France or Britain.
• Hence option (a) is the correct answer.

Q 64.A
• Participatory Guarantee Scheme (PGS) is a process of certifying organic products, which ensures
that their production takes place by laid-down quality standards. Hence statement 1 is correct.
o It is implemented by the Ministry of Agriculture and Farmer Welfare. Hence statement 2 is not
correct.
o The certification is in the form of a documented logo or a statement.
o According to the International Federation of Organic Agriculture Movements (IFOAM), the Bonn-
based global umbrella organisation for the organic agriculture movement, PGSs are “locally focused
quality assurance systems” that “certify producers based on active participation of stakeholders and
are built on a foundation of trust, social networks and knowledge exchange”.
• Advantages of PGS:
o Procedures are simple, documents are basic, and farmers understand the local language used.
o It operates on a peer-based appraisal system, wherein members of the same village act as
appraisers, leveraging their closer proximity for enhanced surveillance. This peer-driven

26 www.visionias.in ©Vision IAS


https://upscpdf.com/ https://upscpdf.com/
https://t.me/visioniastestseries2024
https://upscpdf.com/ https://upscpdf.com/

.
approach not only fosters a more robust monitoring system but also curtails expenses by
eliminating the need for third-party certification. Hence statement 3 is not correct.
o Mutual recognition and support between regional PGS groups ensures better networking for
processing and marketing.
o Unlike the grower group certification system, PGS offers every farmer individual certificates, and the
farmer is free to market his produce independent of the group
• limitations of PGS:
o PGS certification is only for farmers or communities that can organise and perform as a group within
a village or a cluster of contiguous villages, and applies only to farm activities such as crop
production, processing, livestock rearing, and off-farm processing “by PGS farmers of their direct
products”.
o Individual farmers or groups of farmers smaller than five members are not covered under PGS.

Q 65.A
• Sandstone comes under ‘minor mineral’ as per the Mines and Minerals (Development and
Regulation) Act, 1957. Being a minor mineral, its production data by the producers is directly submitted
to respective states. Hence statement 1 is correct.
• Sandstone is a sedimentary rock (not a metamorphic rock) that is made up of sand grains of quartz,
felspar, and other detrital minerals with interstitial cementing material. Its size may vary from 2 mm
to 120 mm. Hence statement 2 is not correct.
• The colour of sandstone may range from dark red to brown, earthy to buff, white, yellow, and
several other shades, which is largely defined by the composition of sand particles and the
cementing material. Whereas the mode of formation decides the thickness of the sandstone bed.
• The reserve estimation has not been considered important because of its abundance and easy availability.
Sandstone in India occurs in Rajasthan (Vindhyan and Trans-Aravalli Formations), Gujarat
(Kachchh and Vadodara district, etc), Madhya Pradesh (The Vindhyan and Satpura Mountains),
Uttar Pradesh (Agra, Mirzapur, Lalitpur, etc district), etc. Hence statement 3 is not correct.

Q 66.C
• The sermon, or khutba, serves as the primary formal occasion for public preaching of the Islamic
tradition. Sermons occur regularly, as prescribed by the teachings of almost all legal schools, at the noon
congregation prayer on Friday, the weekly day of assembly, which it is incumbent upon all free and able
adult male Muslim residents to attend.
• The khutba is believed to have its origins in the practice of the prophet Muhammad, who used to speak
words of exhortation, instruction, or command at gatherings for worship in the mosque, which consisted
of the courtyard of his house in Medina. However, the word khutba with this technical meaning does not
appear in the Quran.
• In medieval India, A Sultan was supposed to rule over a territory on behalf of the Khalifa or
Caliph, who was considered to be the spiritual and temporal head of the Muslims. Both the names
of the Khalifa and the Sultan used to be read in the khutba, (Friday prayers) by the local Imams. In
1526 the Delhi Sultans were replaced by the Mughals, who initially ruled from Agra and later from Delhi
till 1707. Thereafter, the Mughal rule continued only nominally till 1857 when the dynasty ended. The
Mughals did not ask for any investiture but continued to send presents to the Khalifa’s. They also got the
khutba to read in their names. Hence option (c) is the correct answer.

Q 67.C
• The Wildlife Protection Act was amended in 2022, which led to major changes in the Schedules of the
Act.
• According to the amendments:
o Schedule I consists of animals, which are given the utmost protection. E.g., lion, tiger, Great India
Bustard, etc. Hence, statement 2 is correct.
o Schedule III (and not Schedule VI) now includes the protected plant species. E.g., Pitcher plant.
Hence, statement 3 is correct.
o The amendment has added a new Schedule IV for the animals listed in the Appendices of CITES.
Hence, statement 1 is correct.
• CITES: The Convention on International Trade in Endangered Species of Wild Fauna and
Flora (CITES) is an international agreement to which states and regional economic integration
organizations voluntarily adhere.

27 www.visionias.in ©Vision IAS


https://upscpdf.com/ https://upscpdf.com/
https://t.me/visioniastestseries2024
https://upscpdf.com/ https://upscpdf.com/

.
Q 68.A
• The Union Cabinet approved the PM-Surya Ghar: Muft Bijli Yojana with a total outlay of Rs. 75,021
crore for installing rooftop sola. It aims at providing free electricity up to 300 units every month for
one-crore households. Hence, statement 1 is correct.
• “The scheme provides for a subsidy of 60% (not 100%) of the solar unit cost for systems up to two
kilowatts (kW) capacity and 40% of the additional system cost for systems between two to three kW
capacity. Further, the subsidy has been capped at three kW capacity. Hence, statement 2 is not correct.
• This setup reduces dependence on grid-connected electricity, resulting in cost savings. "Net Metering"
is hence the most important component, as it enhances the financial viability of the entire process.
It allows any excess energy generated by the solar panels to be sold back to electricity distribution
companies at a fair price. Hence, statement 3 is not correct.

Q 69.C
• For a long time, scientists have observed that Africa is splitting into two potentially forming a new ocean.
As Africa splits into two pieces, scientists predict the beginning of the rarest natural phenomena.
Researchers believe that in the distant future, the creation of a new ocean may result in the division of
Africa into two pieces. The separation of two significant portions of the continent may eventually lead to
the formation of a new body of water. In millions of years, landlocked nations like Zambia and Uganda
might have their own coasts.
• According to Science Direct, rifting is the tearing apart of a “single tectonic plate into two or more
tectonic plates separated by divergent plate boundaries”. For the last 30 million years, the Arabian
plate has been drifting away from Africa, a process that resulted in the creation of the Red Sea and
the Gulf of Aden. Hence, statement 1 is correct.
• The splitting up of the continent is linked to the East African Rift, a 56-kilometer or 35-mile-long crack
that emerged in Ethiopia’s desert in 2005. Spanning over 3,000km, the East African Rift Valley lies from
the Gulf of Aden in the north towards Zimbabwe in the south. This will set off the formation of a new
sea. The crack was discovered at the border of three tectonic plates – African Nubian, African
Somali, and Arabian – that have already been separating for some time. Hence, statement 2 is
correct.
• It will take millions of years for Africa to be sliced into two unequal parts. The new ocean will take at
least 5 million to 10 million years to form which could eventually give the landlocked countries of
Uganda and Zambia their own coastlines. The smaller continent created by the rift will include countries
such as present-day Somalia and parts of Kenya, Ethiopia, and Tanzania.

28 www.visionias.in ©Vision IAS


https://upscpdf.com/ https://upscpdf.com/
https://t.me/visioniastestseries2024
https://upscpdf.com/ https://upscpdf.com/

.
Q 70.B
• The Multi-Angle Imager for Aerosols (MAIA) mission is a collaborative effort between NASA and
the Italian Space Agency (ASI) designed to investigate the link between air pollution, specifically
fine particulate matter (PM2.5), and its health impacts in selected megacities worldwide. Hence
statement 1 is correct.
• The three-year mission will focus on 11 primary target areas including New Delhi from India.
Hence statement 3 is correct.
• MAIA is a satellite instrument that will collect data to examine the health effects of different types of air
pollution. It will use measurements of sunlight reflecting off airborne particles to determine the
abundance, size, chemical composition, and optical properties of pollutants in the atmosphere. The data
will be collected from sensors on the ground and atmospheric models. These results will then be related to
respiratory and cardiovascular diseases, adverse reproductive outcomes, human birth, death and
hospitalisation records to understand the impacts of contaminated air we breathe.
• MAIA is not a constellation mission. It utilizes a single satellite platform with an advanced imaging
instrument. Hence statement 2 is not correct.
• The observatory will consist of the PLATiNO-2 satellite, which ASI will provide. The observatory’s
science instrument contains a pointable spectropolarimetric camera, which captures digital images at
multiple angles in the ultraviolet, visible, near-infrared and shortwave infrared portions of the
electromagnetic spectrum.

Q 71.A
• The India Gazette, or Calcutta Public Advertiser, was an English-language weekly newspaper
published in Calcutta (now Kolkata), which was then the capital of British India. It was started
by Henry Louis Vivian Derozio. It represents an important part of India's colonial history and media
landscape. Hence pair 1 is not correctly matched.
• Sambad Kaumudi was a Bengali weekly newspaper published from Kolkata in the first half of the
19th century by Ram Mohan Roy. It was a noted pro-reformist publication that actively campaigned for
the abolition of the Sati Pratha. Hence pair 2 is not correctly matched.
• The Hindu Patriot, published in Kolkata, is one of the earliest efforts of the Indian press. The Hindu
Patriot was launched in 1853 under the editorship of Girish Chandra Ghosh, and by 1855 Harish
Chandra Mukherjee had assumed the editorship of the journal. Hence pair 3 is correctly matched.

Q 72.C
• The United Nations Country Team (UNCT) exists in 130 countries, covering all of the 162 countries
where there are United Nations programmes. The United Nations Country Team (UNCT) includes all
the UN entities working on sustainable development, emergency, recovery and transition in programme
countries. Hence statement 1 is correct.
• The UNCT is led by the UN Resident Coordinator, who is the representative of the UN Secretary-
General in a given country. The Resident Coordinator (RC) is the highest ranking representative of the
United Nations development system (UNDS) at the country level with the responsibility to lead United
Nations country teams. Hence statement 2 is correct.
• The UNCT exists in 132 countries, covering all of the 162 countries where there are United Nations
programmes.
• The UN Sustainable Development Group oversees the coordination of development of the UNCT in
162 countries and territories. It also guides, supports and tracks the development of programmes. Hence
statement 3 is correct.

Q 73.A
• Koya Rebellion:
o In 1879-1880 there occurred the Koya rebellion in the eastern Godavari tract of present-day Andhra
Pradesh which also affected some portions of the Malkangiri region of Koraput district in Orissa. It
was led by Tomma Dora, the Koya leader. Hence pair 2 is correctly matched.
o The movement reflected the problems faced by the tribals like the erosion of customary rights over
forests and the exploitation by moneylenders who began to control the life of the Koyas through loans
and land transfers.
o Tomma Dora was hailed by the Koyas as the 'King' of Malkangiri. We find references to the taking
over of a police station at Motu by the rebels. However, very soon after this Dora was shot dead by
the police and the movement collapsed.
29 www.visionias.in ©Vision IAS
https://upscpdf.com/ https://upscpdf.com/
https://t.me/visioniastestseries2024
https://upscpdf.com/ https://upscpdf.com/

.
• Khond Uprisings (1837–56):
o From 1837 to 1856, the Khonds of the hilly tracts extending from Odisha to the Srikakulam and
Visakhapatnam districts of Andhra Pradesh revolted against Company rule.
o Chakra Bisoi, a young raja, led the Khonds, who were joined by the Ghumsar, Kalahandi, and
other tribals to oppose the suppression of human sacrifice, new taxes, and the entry of zamindars into
their areas. Hence pair 1 is not correctly matched
o With Chakra Bisoi’s disappearance, the uprising came to an end. A later Khond rebellion in 1914 in
the Orissa region was triggered by the hope that foreign rule would end and they could gain an
autonomous government.
• The Santhal Rebellions (1833; 1855–56):
o Santhal Rebellion by the Santhals led by Sido and Kanhu (1855– 56; Bihar) against the practices
of zamindars and moneylenders. The rebellion later turned anti-British and was suppressed. Hence
pair 3 is not correctly matched.
o It needs to be noted that the Santhals moved into the Rajmahal area in the late 1770s and early 1780s
from the region of Cuttack, Dhalbhum, Manbhum, Hazaribagh, and Midnapore. They showed a
willingness to give up a nomadic life and lead a settled life. They were ready to clear the forests,
plough the land, and grow crops.
o This suited the British interests; the Company wanted more revenue from land and more crops for
export. The British, therefore, persuaded the Santhals to settle in the foothills of Rajmahal. But a feud
between the Though this chapter is basically about the uprisings before the 1857 Revolt, it cannot be
forgotten that some tribal revolts continued to take place after 1857.

Q 74.C
• According to the World Bank, India's urban population share has grown steadily from 30.9% in 2000
to 44.4% in 2020. Hence statement 1 is correct.
• According to the RBI Monetary Policy Report, Dec 2023, the secondary sector (manufacturing,
construction) offers more formal and organised employment opportunities compared to the primary sector
(agriculture). This attracts people to urban areas where these industries are concentrated. Hence
statement 2 is correct.
• According to the National Sample Survey Office (NSSO), Migration in India Report, high living
costs, work-from-home options, and pollution are driving a trend of migration from big cities to smaller
towns and even rural areas. Hence statement 3 is correct.

Q 75.A
• The Foreigners (Tribunals) Order, 1964 was issued by the Central Government under Section 3 of The
Foreigners Act, 1946. It applies to the whole country. Major amendments in the Foreigners (Tribunals)
Order, 1964 were undertaken in 2013. The last amendment was issued in May 2019. All these orders
apply to the whole country and are not specific to any state.
• Amendment to Foreigners (Tribunals) Order:
o It has empowered district magistrates in all States and Union Territories to set up tribunals
(quasi-judicial bodies) to decide whether a person staying illegally in India is a foreigner or not.
Earlier, the powers to constitute tribunals were vested only with the Centre. Hence statement 1 is not
correct.
o The amended order (Foreigners (Tribunal) Order, 2019) also empowers individuals to approach
the Tribunals. Earlier, only the State administration could move the Tribunal against a suspect.
Hence statement 2 is correct.
• Since NRC work is going on only in Assam, therefore, the aforementioned Order, issued on 30th May
2019 applies only to Assam as of date for all practical purposes. This Amendment Order also provides for
reference by the District Magistrate to the Tribunal for its opinion as to whether the Appellant is a
“foreigner” or not within the meaning of the Foreigners Act, 1946. Since the Foreigners Tribunals
under this order have been established only in Assam and in no other state of the country, this
amendment in effect is going to be relevant only to Assam at present. Hence statement 3 is not
correct.

Q 76.D
• Indian space station:
o The Bharatiya Antariksha Station (BAS) is the official name for the Indian orbital space station, a
project unveiled in 2019 by the then ISRO chief K. Sivan. It was initially supposed to be completed
by 2030, but the Covid pandemic delayed the timeline.
30 www.visionias.in ©Vision IAS
https://upscpdf.com/ https://upscpdf.com/
https://t.me/visioniastestseries2024
https://upscpdf.com/ https://upscpdf.com/

.
o The first module of the space station will be propelled into orbit by 2028, ISRO chairman S.
Somanath said at an event in December 2023. Modules are spacecraft sections, which, once
assembled, form the space station. For example, the International Space Station (ISS), the largest
single structure ever put in space, has over 40 modules at the moment.
• Space stations:
o Currently, two space stations are operational in the Earth’s orbit — the ISS, launched in
2000 as a collaboration between five space agencies in the US, Canada, Japan, Europe, and
Russia, and the Tiangong Space Station, which China launched in 2021. Hence statement 2 is not
correct.
• History of Space stations:
o In 1971, the Soviet Union launched the first-ever space station, Salyut (salute), followed that up
by launching six more improved space stations, up to Salyut 7. Hence statement 1 is not correct.
o Rakesh Sharma, India’s first astronaut to go to space in 1984, spent eight days on board Salyut
7 in a joint operation between ISRO and the Soviet Interkosmos space agency. Salyut 7 finally left
orbit in 1991. Hence statement 3 is not correct.
o Experience gained from the Salyut stations helped the Soviet Union launch the Mir Space Station in
1986. The Mir station remained in orbit till 2001, spending 15 years in space.
o Russian cosmonaut Valeri Polyakov, who spent 438 days on the Mir station, holds the record for the
longest human space mission now. A Salyut-derived module remains in orbit at the core of the
Russian segment of the ISS.
• International Space Station(ISS) Facts:
o An international partnership of five space agencies from 15 countries operates the International Space
Station.
o The space station has been continuously occupied since November 2000.
o An international crew of seven people live and work while traveling at a speed of five miles per
second, orbiting Earth about every 90 minutes.
o Sometimes more are aboard the station during a crew handover.
o In 24 hours, the space station makes 16 orbits of Earth, traveling through 16 sunrises and sunsets.
Q 77.D
• In October 1929 Lord Irwin, the Viceroy stated that it is implicit in the Declaration of 1917 that the
natural issue of India’s progress as there contemplated, is the attainment of dominion status.’ He also
promised a Round Table Conference as soon as the Simon Commission submitted its report. Two days
later, on November 2, 1929, a conference of prominent national leaders issued a ‘Delhi Manifesto’ in
which they demanded that it should be made clear that the purpose of the Round Table Conference
was not to discuss when Dominion Status should be granted but to formulate a scheme for its
implementation. They also demanded that Congress should have majority representation at the
conference; and there should be a general amnesty for political prisoners and a policy of
conciliation.
• Hence both the statements are not correct.

Q 78.B
• India, which accounts for 3 percent of the global solar manufacturing market, is expected to become the
world’s second-largest module manufacturing region by 2025, Wood Mackenzie said in its latest
report. Hence statement 1 is not correct.
• Looking outside China, India is forecasted to overtake Southeast Asia as the second-largest module
production region by 2025, which will be driven by India’s strong PLI incentives. India’s cumulative
module manufacturing nameplate capacity more than doubled from 18GW in March 2022 to 38GW in
March 2023.
• In terms of upcoming PV manufacturing installations, Gujarat is the leading state in India. It
accounts for nearly 57% of all the upcoming PV manufacturing capacity. Some major reasons
manufacturers chose Gujarat for setting up their PV fabrication facilities include cheaper industrial
electricity prices and easy access to ports for imports and exports. Hence statement 2 is correct.
• The report identifies some hurdles holding back the domestic PV manufacturing industry from
realizing its full growth potential, chief among them over-reliance on Chinese imports for upstream
components of PV modules such as polysilicon and ingots/wafers. According to the report, the
sustained reliance on China for its raw material is such that almost all (around 95%) of the upstream PV
manufacturing capabilities are still in China. Another is the dearth of skilled manpower, especially in the
manufacturing of these upstream components, Hence statement 3 is correct.

31 www.visionias.in ©Vision IAS


https://upscpdf.com/ https://upscpdf.com/
https://t.me/visioniastestseries2024
https://upscpdf.com/ https://upscpdf.com/

.
Q 79.B
• Prime Editing is a cutting-edge genome editing technique that enables precise modifications to DNA
sequences by combining a catalytically impaired Cas9 enzyme with an engineered reverse
transcriptase.
• This fusion protein allows for the direct rewriting of specific DNA sequences without requiring double-
strand breaks.
• The key components of Prime Editing include a Prime Editing guide RNA (pegRNA) and a Cas9
nickase variant fused to a reverse transcriptase. The pegRNA contains a target-specific guide sequence
for directing the Cas9 enzyme to the desired genomic location and a template sequence for specifying the
desired edit.
• Prime Editing initiates with the binding of the pegRNA-Cas9 complex to the target DNA site. The Cas9
nickase creates a single-strand nick in the DNA, facilitating the insertion, deletion, or substitution of
nucleotides specified by the template sequence in the pegRNA. The reverse transcriptase then synthesizes
the edited DNA strand based on the template, resulting in precise modifications.
• Prime Editing offers several advantages over traditional CRISPR-Cas9 methods, including greater
precision in editing, reduced risk of off-target effects, and the ability to introduce a wide range of
edits, including insertions, deletions, and base substitutions, without inducing double-strand
breaks. Hence option (b) is the correct answer. Prime Editing can be used to correct disease-causing
mutations in the human genome, engineer crops with desired traits, develop animal models for studying
genetic diseases and advance our understanding of gene function and regulation. Additionally, Prime
Editing may facilitate the development of novel therapeutic interventions for genetic disorders.

Q 80.A
• Arbitrage is the purchase and sale of an asset in order to profit from a difference in the asset's price
between markets. It is a trade that profits by exploiting the price differences of identical or similar
financial instruments on different markets or in different forms. Arbitrage exists as a result of market
inefficiencies and would therefore not exist if all markets were perfectly efficient. Hence option (a) is the
correct answer.
• People who engage in arbitrage are called arbitrageurs such as a bank or brokerage firm. The term is
mainly applied to trading in financial instruments, such as bonds, stocks, derivatives, commodities and
currencies. Arbitrage has the effect of causing prices of the same or very similar assets in different
markets to converge.

Q 81.B
• Indian Ocean Rim Association (IORA) is an intergovernmental organisation established to promote
economic cooperation and regional integration among countries bordering the Indian Ocean. IORA
member states work on various initiatives related to trade, investment, and sustainable development in the
Indian Ocean region (IOR).
• The objectives of the Association are underpinned by the principle of open regionalism as below:
o To promote sustainable growth and balanced development of the region and Member States.
o To focus on those areas of economic co-operation which provide maximum opportunities for
development, shared interest and mutual benefits.
o To promote liberalization, remove impediments and lower barriers towards a freer and enhanced flow
of goods, services, investment, and technology within the Indian Ocean Rim.
• Membership is open to all sovereign states of the Indian Ocean Rim willing to subscribe to the
principles and objectives of the Charter.
• Current Member States: Australia, Bangladesh, Comoros, France/Reunion, India, Indonesia, Iran,
Kenya, Madagascar, Malaysia, Maldives, Mauritius, Mozambique, Oman, Seychelles, Singapore,
Somalia,South Africa, Sri Lanka, Tanzania, Thailand, United Arab Emirates and Yemen.
• Dialogue Partners:China, Egypt, Germany, Italy, Japan, Republic of Korea, Russia, Turkey, the United
Kingdom and the United States of America.
• Specialized Agencies:
o The Regional Centre for Science and Technology Transfer (RCSTT) based in Tehran, Iran.
o The Fisheries Support Unit (FSU) based in Muscat, Oman.
• Two Observers:
o The Indian Ocean Research Group (IORG)
o The Western Indian Ocean Marine Science Association (WIOMSA)

32 www.visionias.in ©Vision IAS


https://upscpdf.com/ https://upscpdf.com/
https://t.me/visioniastestseries2024
https://upscpdf.com/ https://upscpdf.com/

.
• The Secretariat is hosted by the Government of the Republic of Mauritius which is based in Cyber
City, Ebène, Mauritius.
• Hence option (b) is the correct answer.

Q 82.C

• Hence option (c) is the correct answer.

Q 83.C
• ISRO's CE-20 rocket engine:

33 www.visionias.in ©Vision IAS


https://upscpdf.com/ https://upscpdf.com/
https://t.me/visioniastestseries2024
https://upscpdf.com/ https://upscpdf.com/

.
• It has been designed and developed by the Liquid Propulsion Systems Centre (LPSC), a subsidiary
of ISRO.
• ISRO has used three cryogenic engines over the years: KVD-1, CE-7.5, and CE-20.
• The last two are India-made, although the design of the CE-7.5 is based on the KVD-1, which Russia (as
the Soviet Union) supplied to India in the early 1980s. The GSLV Mk II launch vehicle uses CE-7.5
engines to power the third stage of its ascent. Thus CE-20 is the first indigenously built cryogenic
engine. Hence statement 2 is correct.
• The operation of a cryogenic engine requires a cryopump, a device to trap and cool the hydrogen and
oxygen to liquid form; special storage tanks; and turbopumps to move the cooled fuel and oxidizer to the
engine. The CE-7.5 engine uses the staged combustion cycle. Here, a small amount of the fuel is
combusted in a pre-burner. The resulting heat is used to drive the turbine that powers the turbopump.
• The CE-20 engine uses the gas-generator cycle, which discards the exhaust from the pre-burner instead
of sending it to the combustion chamber. This reduces fuel efficiency but, importantly for ISRO, makes
the CE-20 engine easier to build and test. It uses liquid hydrogen, which is the most efficient fuel for
rockets. Hence statement 1 is correct.
• ISRO has also dropped its vernier thrusters in favor of allowing the engine’s nozzle to make small
rotations — or gimbal — to adjust the rocket’s flight path. As a result, while the CE-7.5 engine is lighter
and sports higher fuel-use efficiency, the CE-20 engine achieves a higher maximum thrust (~200
kilonewtons v. 73.5 kilonewtons) with a shorter burn duration.
• ISRO has recently accomplished a major milestone in the human rating of the CE20 cryogenic
engine, which powers the cryogenic stage of the human-rated LVM3 launch vehicle for Gaganyaan
missions, with the completion of the final round of ground qualification tests. Hence statement 3 is
correct.

Q 84.D
• A few references show that the Rig Vedic period had knowledge of agriculture and practiced it to
supplement their food requirements. They produced yava (modern jau or barley), which was rather a
generic word for cereals (food grains). Apart from cattle-rearing and small-scale cultivation, people were
engaged in many other economic activities. Hunting, carpentry, tanning, weaving, chariot-making, metal
smeltry, etc. were some such activities. The products of these activities were exchanged through barter.
However, cows were the most favored medium of exchange. The priests received cows, horses and gold
ornaments as fees for performing sacrifices.
• Hence option (d) is the correct answer.

Q 85.D
• Recently, Vanadium, a critical raw material for many industrial applications, has been found in sediment
samples collected from the Gulf of Khambhat, which opens into the Arabian Sea off Alang in Gujarat.
What makes this find a significant one is that the war mineral, used to strengthen steel and make batteries,
is scarce in India.
• Rarely found in its pure form naturally, vanadium is present in over 55 different minerals, which makes its
production costly. At the Gulf of Khambhat, it has been found in a mineral called titanomagnetite, which
is formed when molten lava cools rapidly. Vanadium is a critical raw material for strategic sectors
such as defence and aerospace. For instance, vanadium-containing alloys of titanium and aluminum
are used in jet engine components and high-speed airframes.
• It has good structural strength and a low neutron cross-section for nuclear fission reactions, making
it useful for nuclear applications.
• Besides these, the metal is also used for storing energy and in making critical electronic components.
• It is used to make alloys that are resistant to corrosion, wear, and high temperatures.
• It is also used to make vanadium redox flow batteries, which are promising for large-scale energy
storage.
• Traces of the metal have so far been found in Arunachal Pradesh, Karnataka, Odisha and Maharashtra.
• Hence, option (d) is the correct answer.

Q 86.A
• Basholi Paintings are known for their miniature art style from the Kathua district in Jammu and have
received the Geographical Indication (GI) Tag. Hence statement (a) is not correct.
• It is the first independent GI-tagged product from the Jammu region. Hence statement (b) is
correct.
34 www.visionias.in ©Vision IAS
https://upscpdf.com/ https://upscpdf.com/
https://t.me/visioniastestseries2024
https://upscpdf.com/ https://upscpdf.com/

.
• It is considered the first school of Pahari paintings. Pahari denotes ‘hilly or mountainous’ in origin.
Pahari Schools of Painting include towns, such as Basohli, Guler, Kangra, Kullu, Chamba, Mankot,
Nurpur, Mandi, Bilaspur, Jammu, and others in the hills of western Himalayas, which emerged as centers
of painting from the seventeenth to the nineteenth century. Hence statement (c) is correct.
• The first and most dramatic example of work from the hill states is from Basohli. Under Kirpal Pal, who
ruled from 1678 to 1695, Basohli developed a distinctive and magnificent style.
o It is characterized by a strong use of primary colors and warm yellows—filling the background and
horizon. The most significant characteristic of the Basohli painting is the use of small, shiny green
particles of beetle wings to delineate jewelry and simulate the effect of emeralds.
o The most popular theme of Basohli painters was the Rasamanjari of Bhanu Datta. Bhagvata
Purana and Ragamala were other popular themes.
o These are painted on paper, cloth, or wood using natural pigments and dyes. Hence statement (d)
is correct.

Q 87.B
• In a general sense, any Bill that relates to revenue or expenditure is a Financial Bill. A Money Bill is also
a specific type of Finance Bill, that must deal only with matters specified in Article 110 (1) (a) to (g).
• More specifically, Article 117 of the Constitution deals with the special provisions relating to Financial
Bills. Article 117 (1) indicates that a Bill that makes provision for any of the matters specified in clauses
(a) to (f) of Article 110 (1) can be introduced or moved only on the President’s recommendation and
cannot be introduced in the Rajya Sabha. Examples of this first category of Financial Bills are Money
Bills and other Financial Bills originating solely in the Lok Sabha.
• The second category of Finance Bills is dealt with under Article 117 (3) of the Constitution. Such Bills are
more like ordinary Bills. The difference between this kind of Financial Bill and an ordinary Bill is that if
the former is enacted, it will involve expenditure from the Consolidated Fund of India and cannot be
passed by either House unless the President has recommended its consideration. In all other respects, such
Financial Bills are just like ordinary Bills, and can even be introduced in the Rajya Sabha, amended by it,
or be subjected to deliberation by both Houses in a joint sitting.
• A Financial Bill becomes a Money Bill when it exclusively falls under one of the seven heads listed
under Article 110(1), which defines Money Bills. Moreover, a Money Bill is a Financial Bill that is
certified by the Speaker. Hence, statement 1 is not correct.
• Article 110 defines a “Money Bill” as one containing provisions dealing with taxes, regulation of the
government’s borrowing of money, and expenditure or receipt of money from the Consolidated Fund of
India, among others, whereas Article 109 delineates the procedure for the passage of such a Bill and
confers an overriding authority on the Lok Sabha in the passage of Money Bills.
• A major difference between money and Financial Bills is that while the latter has the provision of
including the Rajya Sabha’s (Upper House) recommendations, the former does not make their inclusion
mandatory. The role of the Rajya Sabha in passing Money Bills is restricted. Such Bills can originate
only in the Lok Sabha. After being passed by the Lok Sabha, Money Bills are sent to the Rajya Sabha for
its recommendations. Within 14 days, the Upper House must submit the Bill back to the Lower House
with its non-binding recommendations. If the Lok Sabha rejects the recommendations, the Bill is deemed
to have passed by both Houses in the form in which it was passed by the Lok Sabha without the
recommendations of the Rajya Sabha. The Lok Sabha has the right to reject the Rajya Sabha’s
recommendations when it comes to Money Bills.
• Meanwhile, ordinary Bills and other Financial Bills still require the agreement of both Houses of
Parliament to ensure their passage. They can very well be rejected or amended by the Rajya Sabha,
unlike Money Bills. Also, all other Financial Bills, separate from Money Bills, must go through the
rigours of all stages in the Rajya Sabha as ordinary Bills. This means that while the President can
summon a joint sitting of both Houses to resolve differences over a deadlock in passing an ordinary
Bill, there is no provision for a joint sitting for differences over a Money Bill. Hence, statement 2 is
correct.
Q 88.B
• Alipore Bomb Conspiracy Case (1908):
o Background:
▪ Revolutionary activities began in Bengal in 1902, notably with the formation of secret societies
like Anushilan Samity.
▪ Anushilan Samity, led by Jatindernath Banerjee and Barindarkumar Ghose, was involved
in revolutionary activities.

35 www.visionias.in ©Vision IAS


https://upscpdf.com/ https://upscpdf.com/
https://t.me/visioniastestseries2024
https://upscpdf.com/ https://upscpdf.com/

.
o Incident:
▪ Conspiracy to kill Douglas Kingsford, known for harsh treatment of swadeshi agitators.
▪ Khudiram Bose and Prafulla Chaki were tasked with the assassination.
▪ A bomb was thrown on a carriage on April 30, 1908, killing two English women instead.
▪ Prafulla Chaki committed suicide; Khudiram Bose was arrested and hanged.
o Trial:
▪ Aurobindo Ghosh, along with Barindra Kumar Ghosh and thirty-five others, was tried in
the case. Hence pair 2 is not correctly matched.
▪ Other Names: Also referred to as the Maniktala bomb conspiracy or the Murarupukur conspiracy.
• Delhi Lahore Conspiracy Case (1912)
o Background: Also known as the Delhi Conspiracy Case or Hardinge Bomb Case.
o Incident:
▪ An assassination attempt on Viceroy Lord Hardinge led by revolutionaries, led by Rash
Behari Bose and Sachin Sanyal. Hence pair 1 is not correctly matched.
▪ Basant Kumar Biswas, Amir Chand, and Avadh Behari were convicted and executed for the
attempt.
• Kanpur Conspiracy Case (1924):
o Background:
▪ Muzaffar Ahmed, Shaukat Usmani, Nalini Gupta, and S. A. Dange were sent to jail for four
years for communist activities. Hence pair 4 is correctly matched.
o Outcome: Trial and imprisonment raised awareness about communist activities in India.
• Nashik Conspiracy(1909):
o Background:
▪ Mitra Mela and Abhinav Bharat (1899-1904)
▪ Savarkar and his brother organized Mitra Mela, a secret society, in 1899.
▪ It was merged with Abhinav Bharat in 1904, inspired by Mazzini's 'Young Italy.'
▪ Nasik, Poona, and Bombay emerged as centers of bomb manufacture.
▪ They planned a conspiracy to overthrow the British government in India employing armed
revolution.
▪ A.M.T. Jackson, Collector of Nasik, was killed by Anant Lakshman Kanhere in 1909 as part
of the conspiracy.
o Conspiracy Trial and Sentencing (1909)
▪ Thirty-eight people were arrested for conspiracy to overthrow British rule through armed
revolution.
▪ Savarkar, along with his two brothers, is identified as the brain, leader, and moving spirit of the
conspiracy.
▪ Savarkar was sentenced to transportation for life and forfeiture of all property. Hence pair
3 is correctly matched.
• Other Conspiracy Cases:
o Meerut Conspiracy Case (1929):
▪ Background:
❖ The most famous communist conspiracy case by the British Government.
▪ Incident: 31 labor leaders, including three Englishmen, were arrested for organizing an Indian
railway strike.
o Central Assembly Bomb Case (1929) & Lahore Conspiracy Case (1931):
▪ Background:
❖ Bhagat Singh and Batukeshwar Dutt threw bombs at the Central Legislative Assembly in
Delhi on April 8, 1929.
❖ Bhagat Singh, Rajguru, Sukhdev, Jatindra Nath Das, and others were arrested and tried for the
murder of Saunders.
▪ Outcome: Bhagat Singh, Rajguru, and Sukhdev were hanged on March 23, 1931, in Lahore Jail,
chanting slogans against British imperialism.
o Kakori Conspiracy (1925)
▪ Background: Train robbery near Kakori, Lucknow, organized by the Hindustan Republican
Association (HRA) against the British government.
▪ Incident: Led by Ram Prasad Bismil and supported by Ashfaqulla Khan, Rajendra Lahiri,
Chandrashekhar Azad, and others.

36 www.visionias.in ©Vision IAS


https://upscpdf.com/ https://upscpdf.com/
https://t.me/visioniastestseries2024
https://upscpdf.com/ https://upscpdf.com/

.
Q 89.A
• A public good is a commodity or service that every member of a society can use without reducing its
availability to all others. Typically, a public good is provided by a government and funded through taxes.
• A public good has two characteristics:
o Non-rivalry: This means that when a good is consumed, it doesn’t reduce the amount available to
others.
o Non-excludability: This occurs when it is not possible to provide a good without it being possible for
others to enjoy.
• Examples of a public good include a town road, park, or school. National Defence is a public good. As
the government provides for the public goods and amenities, it secures finances to create the same
through the taxes collected from the public (citizens of the country) who ultimately bear the cost. In
the process, the public loses a chunk of their savings that could have been used for other
purposes. Thus there is always an opportunity cost involved. Hence statement 1 is correct.
• A free good is a good that is not scarce and is available without limit. Free goods have zero opportunity
cost to society. Free goods, such as water, air, and sunshine, are considered to have zero opportunity cost
because theoretically no sacrifice is made to obtain them. Public goods do not have zero opportunity
cost. Hence statement 2 is not correct.
o Opportunity cost is the loss of benefit or value that would have been derived from an option that is not
chosen.

Q 90.D
• More than 65% of India's electricity generation capacity comes from thermal power plants, with 85% of
the country's thermal power generation being coal-based. The ten biggest thermal power stations
operating in India are all coal-fired, with five of them owned and operated by state-run National Thermal
Power Corporation (NTPC).
• Some of India's biggest thermal power plants by installed capacity are
o Vindhyachal Thermal Power Station, Madhya Pradesh
▪ Installed capacity of 4,760MW
o Mundra Thermal Power Station, Gujarat
▪ 4,620MW
o Sasan Ultra Mega Power Plant, Madhya Pradesh
o Tiroda Thermal Power Plant, Maharashtra
o Talcher Super Thermal Power Station, Odisha
o Rihand Thermal Power Station, Uttar Pradesh
o Sipat Thermal Power Plant, Chhattisgarh
o Chandrapur Super Thermal Power Station, Maharashtra
o NTPC Dadri, Uttar Pradesh

• Hence option (d) is the correct answer.


37 www.visionias.in ©Vision IAS
https://upscpdf.com/ https://upscpdf.com/
https://t.me/visioniastestseries2024
https://upscpdf.com/ https://upscpdf.com/

.
Q 91.B
• Dibru-Saikhowa National Park is a national park located in Dibrugarh and Tinsukia districts, Assam,
India. It was designated a Biosphere Reserve in July 1997 with an area of 765 km2, including a core area
of 340 km2 and a buffer zone of 425 km2.
o It is located at about 12 km north of Tinsukia town at an average elevation of 118 m ranging
from 110 to 126 m. The park is bounded by the Brahmaputra and Lohit Rivers in the north and
Dibru River in the south. Hence pair 1 is correctly matched.
o It mainly consists of moist mixed semi-evergreen forests, moist mixed deciduous forests, canebrakes
and grasslands. It is the largest salix swamp forest in north-eastern India, with a tropical monsoon
climate with a hot and wet summer and cool and usually dry winter.
o Annual rainfall ranges from 2,300 to 3,800 mm. It is a haven for many endangered species and rich in
fish diversity.
• The Great Himalayan National Park Conservation Area is located in the western part of the Himalayan
Mountains in the northern Indian State of Himachal Pradesh.
o It includes the upper mountain glacial and snow melt water source origins of the westerly
flowing Jiwa Nal, Sainj and Tirthan Rivers and the north-westerly flowing Parvati River which are
all headwater tributaries to the River Beas and subsequently, the Indus River.
▪ Sainj River is one of the major tributary of river Beas with an altitudinal range between 900
to 5800 meters.
o Hence pair 2 is correctly matched.
o The property lies within the ecologically distinct Western Himalayas at the junction between two of
the world’s major biogeographic realms, the Palearctic and Indomalayan Realms.
o The Great Himalayan National Park Conservation Area displays distinct broadleaf and conifer forest
types forming mosaics of habitat across steep valley side landscapes.
• Silent Valley Park is located in the South-western corner of Nilgiris. The whole Park is a roughly
rectangular tableland closed on all sides. It has high and continuous ridges along its entire east, north and
northeast borders and a somewhat lower ridge along the entire western and southern border.
o A perennial river named Kunthipuzha is passing through the western side of the park, from
north to south direction finally merging in to Bharathapuzha. Hence pair 3 is not correctly
matched.
▪ Along its entire length, the plateau slopes toward the bed of Kunthipuzha river, which divides it to
two halves.
o The terrain is generally undulating with steep escarpments and many hillocks. The elevation ranges
from 900 M to 2,300 M above MSL with the highest peak at 2,383 M (Anginda peak).
• The Kabini river flows through the Wayanad Wildlife Sanctuary, Mudumalai National Park, Bandipur
National Park and Nagarhole National Park, which are part of the Nilgiri Biosphere Reserve, a UNESCO
World Heritage.

Q 92.D
• Law is basically a set of rules that are created and enforced by a particular country or community through
social or governmental institutions to regulate the actions of its members. There are five types of legal
systems i.e. civil law; common law; customary law; religious law and mixed law.
• Common law: The Common Law also known as case law or Judicial precedent or judge-made law
is a section of law that is derived from the judicial decision of courts and similar tribunals. As the
name suggests it is common to all. The example set by higher courts is binding on cases tried in
lower courts. Lower courts can also choose to overturn the precedent, but this rarely occurs.
Judgments that include the following are not considered as a precedent:
o The judgment that is not expressed.
o The judgment is not founded on reasons.
o An Obiter Dicta of a case is not binding as it has a persuasive value.
o Judgments made on Per Incuriam cannot be used as precedent. The literal meaning of per
incuriam is resulting from ignorance.
o Judgments where point of law or particular question of law was not consciously determined are
also not binding.
o The Court's observations on the facts of the case are not binding. Hence, option (d) is the
correct answer.
• Criminal law: Criminal law is enforced by the police. Cases like murder, rape, assault, robbery are
dealt under Criminal Law. Offenses that are committed against any individual but are seen as being
against everybody, even though it does not, come under Criminal law. For example, if the house is
38 www.visionias.in ©Vision IAS
https://upscpdf.com/ https://upscpdf.com/
https://t.me/visioniastestseries2024
https://upscpdf.com/ https://upscpdf.com/

.
burgled then the theft is against the individual, but it threatens all house owners because they might have
burgled their houses. Because the view is taken that everybody is threatened by the crime this law is dealt
with by the public services and not by private investigators.
• Civil law: Civil law is law that looks at actions that are not the crime. It is a section of law dealing
with disputes between organizations and individuals. It covers different areas such as defamation,
custody of children, right to education, divorce, trade union membership, property disputes,
ownership issues, copyrights, insurance claims, etc. For example, a person by force took over someone
else property without his/her permission and not vacate it or one company sued another over a trade
dispute or car crash victim claims from the driver for loss or injury sustained in an accident.
• Statutory law: Statute or Statutory Law is a law established by an act of the legislature that is signed by
the executive or legislative body. For state law, the acts are passed by the state legislature and signed by
the state governor. In rare circumstances, the executive (President or governor) may refuse to sign the bill
or reject it, which is known as a "veto." A bill is proposed in the legislature and voted upon. For example,
if you are given a citation for violating the speed limit you have broken a vehicle and traffic law.

Q 93.C
• Phytoremediation basically refers to the use of plants and associated soil microbes to reduce the
concentrations or toxic effects of contaminants in the environment.
• Phytoremediation is widely accepted as a cost-effective environmental restoration technology. .
• Phytoremediation is an alternative to engineering procedures that are usually more destructive to the soil.
• Phytoremediation is, however, limited to the root-zone of plants. Also, this technology
has limited application where the concentrations of contaminants are toxic to plants.
• A hyperaccumulator is a plant capable of growing in soil or water with high concentrations of
metals, absorbing these metals through their roots, and concentrating extremely high levels of metals
in their tissues.
• Phytoremediation can be used to remediate contaminated sites, such as industrial areas, mining sites
etc.
• Phytoremediation is not a fast process as it takes time for a plant to grow and absorb heavy metals
etc. Hence option (c) is the correct answer.

Q 94.C
• The causes of aridity, which results in the formation of deserts:
o Atmospheric High-Pressure Zones (Hadley Cells):
▪ Earth's atmospheric circulation patterns, driven by the sun's energy and the rotation of the Earth,
create areas of high pressure near the equator known as Hadley Cells.
▪ In these regions, air rises, cools, and releases moisture, leading to precipitation. However, at
approximately 30 degrees latitude in both hemispheres, the descending air warms, causing it to
hold less moisture, resulting in arid conditions known as the horse latitudes.
▪ Examples:
❖ The Saharan and Arabian deserts lie mainly within the Tropics. They are hot deserts produced
by descending air on the poleward side of Hadley cells, producing a belt of fairly permanent
high pressure. Farther north, the deserts of Central Asia are also caused by persistent high
pressure, but they are well clear of the Tropics and much cooler.
o Continentality (Distance from Oceans):
▪ Areas located far from oceans or large water bodies experience aridity due to continentality.
Moisture-laden air masses originating from the ocean lose their moisture content as they travel
over land, resulting in reduced precipitation inland.
▪ This phenomenon is particularly evident in areas deep within continents, where air currents have
traversed vast land distances, leading to arid conditions.
o Example:
▪ The Gobi Desert, located in China and Mongolia, is primarily arid due to its distance from
the ocean, resulting in continentality.
▪ It is far from any significant moisture sources, and the winds lose moisture as they travel over
land before reaching the desert interior. Hence pair 1 is correctly matched.
o Rainshadow Effects:
▪ Mountains can create rain shadows, where moist air is forced to rise over the windward side of the
mountain, cooling and releasing precipitation.

39 www.visionias.in ©Vision IAS


https://upscpdf.com/ https://upscpdf.com/
https://t.me/visioniastestseries2024
https://upscpdf.com/ https://upscpdf.com/

.
▪ On the leeward side, the descending air warms, resulting in decreased moisture and arid
conditions. Deserts formed by rainshadow effects are common in regions like the western United
States and parts of Chile.
o Example:
▪ The Mojave Desert, located in the southwestern United States, including parts of California,
Nevada, Utah, and Arizona, is primarily a rainshadow desert. Hence pair 3 is correctly
matched.
▪ It lies in the rain shadow of the Sierra Nevada and other mountain ranges, which block moisture-
laden air masses, resulting in arid conditions in the desert region.
o Coastal Cooling:
▪ Coastal areas influenced by cold ocean currents experience coastal cooling, where air reaching the
coast loses moisture due to chilling over the cold water. As the air moves inland, it remains dry,
leading to arid conditions along the coastlines.
▪ Example:
❖ The Atacama Desert, situated along the western coast of South America in Chile, is
influenced by coastal cooling. Cold ocean currents along the coastline chill the air, leading to
low humidity and minimal precipitation in the region. Hence pair 2 is correctly matched.

Q 95.C
• Ghol or Black spotted Crocker: Gujarat declared the Ghol species as the state fish at the Global
Fisheries Conference India 2023. It is called a fisherman’s lottery. Its swim bladders are highly priced
because of their medicinal and commercial values. Hence, option 1 is correct.
• Baobabs: It is also known as the upside-down tree. The baobab is a prehistoric species that predates both
mankind and the splitting of the continents over 200 million years ago. Distribution: There are 9 species
of baobab tree. Two are native to mainland Africa, six to Madagascar, and one to Australia. Mandu, in the
Dhar district of Madhya Pradesh, is perhaps the only place in India where baobab trees are found in
abundance, with an estimated 1,000 trees in the periphery of Mandu town.They occur in seasonally arid
areas, and are deciduous, shedding their leaves during the dry season. Baobabs store water in the
trunk to endure harsh drought conditions. Hence, option 2 is not correct.
• Eurasian Otter: Recently, an Eurasian otter was found in the Chinnar Wildlife Sanctuary in
Kerala.Features: Shy, nocturnal, good swimmer and fish catcher etc. Habitat: Semi-aquatic, lives in
water bodies, marshes, swamp forests, coastal areas etc. Hence, option 3 is correct.
• Fish Mint: It is a herbal plant full of medicinal properties and grown on the ground with wide leaves.
Other Names: It has many names, such as Bishop’s Weed, Chinese Lizard Tail, Fish Wort, Fish Leaf,
Rainbow Plant and Chameleon Plant. Distribution: Its native range extends from the Himalayan foothills
through Southeast Asia, China, Korea and Japan. Features: Has a fish-like taste and smell. Hence, the
name, fish mint. Habitat: It thrives in wet locations and can even grow partially submerged in water.
Hence, option 4 is correct.

Q 96.B
• Lachit Borphukan, who was born on November 24, 1622, was Momai Tamuli Borbarua's youngest child.
He was an excellent military leader who knew the Brahmaputra valley's geography and the nearby hills
like the back of his hand. He was also given administrative, judicial, and military duties as one of the five
Borphukans of the Ahom kingdom by King Charadhwaj Singha. Borphukan was one of the five patra
mantris (councillors) in the Ahom kingdom. Hence statement 1 is not correct.
• Lachit raised the army and preparations were completed by the summer of 1667. His army successfully
retook Guwahati from the Mughal forces. Emperor Aurangzeb after being informed of the defeat at
Guwahati sent an expeditionary force from Dhaka under Ram Singh. Due to the numerical and
technological inferiority of the Ahom forces, Lachit resorted to guerilla tactics which successfully
withered away from the Mughal army. Hence statement 2 is correct.
• The Battle of Saraighat was a decisive victory for the Ahom Kingdom under Lachit Borphukan's
command. Despite being outnumbered, Borphukan's innovative tactics and guerrilla warfare techniques
led to a stunning defeat of the Mughal forces, halting their eastward expansion. Lachit Borphukan was
victorious. The Mughals were forced to retreat from Guwahati. The Mughals were pursued to the Manas
river, the Ahom kingdom's western boundary. Hence statement 3 is correct.

40 www.visionias.in ©Vision IAS


https://upscpdf.com/ https://upscpdf.com/
https://t.me/visioniastestseries2024
https://upscpdf.com/ https://upscpdf.com/

.
Q 97.C
• Recent context: Three medicinal plant species found in the Himalayas have made it to the IUCN
Red List of Threatened Species. Meizotropis pellita (Patwa) has been assessed as ‘critically
endangered’, Fritilloria cirrhosa (Himalayan fritillary) as ‘vulnerable’, and Dactylorhiza hatagirea
(Salampanja) as ‘endangered’.
• Meizotropis pellita:
o Commonly known as Patwa, is a perennial shrub with a restricted distribution that is endemic to
Uttarakhand.
o The species is threatened by deforestation, habitat fragmentation and forest fires.
o The essential oil extracted from the leaves possesses strong antioxidants and can be a promising
natural substitute for synthetic antioxidants in pharmaceutical industries. Hence, it is a Medicinal
plant species. Hence option 1 is correct.
• Fritillaria cirrhosa:
o Commonly known as Himalayan fritillary, is a perennial bulbous herb.
o In China, the species is used for the treatment of bronchial disorders and pneumonia and the plant is
also a strong cough suppressant. Hence, it is a Medicinal plant species. Hence option 2 is correct.
• Dactylorhiza hatagirea:
o Commonly known as Salampanja, is threatened by habitat loss, livestock grazing, deforestation, and
climate change.
o It is extensively used in Ayurveda, Siddha, Unani and other alternative systems of medicine to cure
dysentery, gastritis, chronic fever, cough and stomach aches. Hence, it is a Medicinal plant species.
Hence option 3 is correct.

Q 98.A
• The process of Steel Slag Road:
o The slag is generated from a steel furnace burning at around 1,500-1,600 degrees centigrade in the
form of molten flux material as an impurity.
o The molten material is poured into the slag pits for cooling as per the customized procedure and
further processed to develop stable steel slag aggregates, with better material properties in place of the
natural aggregate commonly used in road constructions.
• Steel slag road in India:
o Surat becomes the first city in India to implement a processed steel slag road project. Hence
statement 3 is correct.
o Joint venture project involving CSIR, CRRI, Ministry of Steel, NITI Ayog, and ArcelorMittal-Nippon
Steel.
• Construction Cost Comparison:
o The construction cost of the processed steel slag road is 30% cheaper than roads built from
natural aggregates. Hence statement 1 is not correct.
o Financial viability due to cost-effectiveness compared to traditional road construction methods.
• Thickness of Road Layers:
o Processed steel slag road requires 30% less thickness compared to regular roads. Hence
statement 2 is not correct.
o Achieves the same level of durability with reduced thickness due to superior material properties of
steel slag aggregates.
• Other Features:
o The steel slag road construction has a lower carbon footprint compared to roads built using natural
aggregates.
o The effect of high temperatures on the steel slag road is insignificant. The steel slag melts at over 200
degrees Celsius, while the temperature during peak summer in India is not above 45 degrees Celsius.
Moreover, the upper layer is made up of bitumen layers,

Q 99.B
• Viceroy Lord Irwin was the Viceroy of India from 3 April 1926 to 18 April 1931.
• The Simon commission report, Nehru Report, All Parties Conference in 1928, Jinnah's 14 points, the
civil disobedience movement, Round Table Conferences, and some other problems had made his regime
extremely stressful.
• At one stage, Irwin had all the leaders of the Congress behind the bar as a strategy and then at one stage
had opened negotiations with Gandhi alone. The strategy worked. An agreement called Gandhi Irwin Pact
was reached (on 5 March 1931).
41 www.visionias.in ©Vision IAS
https://upscpdf.com/ https://upscpdf.com/
https://t.me/visioniastestseries2024
https://upscpdf.com/ https://upscpdf.com/

.
• The All Parties Conference in 1928 included the All India Liberal Federation, All India Muslim
League, Sikh Central League and others. The Conference, on 19 May 1928, constituted a committee
to draft the Constitution.
• On April 1, 1937, Burma formally split from India. Archibald Cochrane became its first governor
and a Council of Ministers took office, ending a century of association.
o British partitioned Burma from India in 1937 in order to weaken the Burmese nationalist movement.
After World War II, under the leadership of U Aung San, this movement reached its pinnacle, and
Burma gained independence on January 4, 1948.
• The historic Dandi March played a role of great significance in our freedom struggle.
o Gandhiji's salt march, also known as the Salt Satyagraha, began on March 12, 1930, from his
ashram in Sabarmati.
• Hence option (b) is the correct answer.

Q 100.D
• Mauryan Empire (324-187 BC)
• Mauryan Kingdoms was succeeded by
o Sunga (181-71 BC)
o Kanva (71-27BC)
o Satavahanas (235-100BC)
o Indo-Greeks, Parthians (180BC-45AD)
o Sakas (90BC-150AD)
o Kushanas (78AD)
• Sangam Age (300 BC – 300 AD)
o Chola
o Cheras
o Pandyas
• Gupta Empire (300AD – 800AD)
• Post-Gupta or Contemporary Gupta
• After the fall of the Gupta Empire, important centers of power arose:
o Vardhana Dynasty (Hence point 2 is not correct.)
o Mukharis
o Hunas
o Pushyabhutis
o Gaudas
o Varman
o Maitrakas
• Also, Rajputs, Senas, and Chauhans succeeded later.
• The Pala empire was founded by Gopala, probably in AD 750. Pala dynasty ruled from the 8th to
the 12th century. (Hence point 5 is not correct.)
• Hence option (d) is the correct answer.

Copyright © by Vision IAS


All rights are reserved. No part of this document may be reproduced, stored in a retrieval system or
transmitted in any form or by any means, electronic, mechanical, photocopying, recording or otherwise,
without prior permission of Vision IAS.

42 www.visionias.in ©Vision IAS


https://upscpdf.com/ https://upscpdf.com/
https://t.me/visioniastestseries2024

You might also like